Exam 1

Ace your homework & exams now with Quizwiz!

A biophysical profile has been completed on a pregnant woman. The nurse interprets which score as normal? A) 9 B) 7 C) 5 D) 3

A

A woman in the 34th week of pregnancy says to the nurse, "I still feel like having intercourse with my husband." The woman's pregnancy has been uneventful. The nurse responds based on the understanding that: A) It is safe to have intercourse at this time. B) Intercourse at this time is likely to cause rupture of membranes. C) There are other ways that the couple can satisfy their needs. D) Intercourse at this time is likely to result in premature labor.

A

A client with a 28-day cycle reports that she ovulated on May 10. The nurse would expect the client's next menses to begin on: A)May 24 B)May 26 C)May 30 D)June 1

A For a woman with a 28-day cycle, ovulation typically occurs on day 14. Therefore, her next menses would begin 14 days later, on May 24.

Leopold's maneuvers steps

- Maneuver 1: What fetal part (head or buttocks) is located in the fundus (top of the uterus)? - Maneuver 2: On which maternal side is the fetal back located? (Fetal heart tones are best auscultated through the back of the fetus.) - Maneuver 3: What is the presenting part? - Maneuver 4: Is the fetal head flexed and engaged in the pelvis?

- Signs that the placenta is separating include:

- a firmly contracting uterus - a change in uterine shape from discoid to globular ovoid - a sudden gush of dark blood from the vaginal opening - and lengthening of the umbilical cord protruding from the vagina.

The extent of the laceration is defined by depth:

- a first-degree laceration extends through the skin - a second-degree laceration extends through the muscles of the perineal body - a third-degree laceration continues through the anal sphincter muscle - a fourth-degree laceration also involves the anterior rectal wall.

A client in labor is administered lorazepam to help her relax enough so that she can participate effectively during her labor process rather than fighting against it. For which adverse effect of the drug should the nurse monitor? A. Increased sedation B. Newborn respiratory depression C. Nervous system depression D. Decreased alertness

- increase sedation is an adverse effect of lorezapam - diazepam and midazolam cause CNS depression - opiods cause newborn respiration depression

A pregnant woman admitted to the labor and birth suite undergoes rapid HIV testing and is found to be HIV-positive. Which of the following would the nurse expect to include when developing a plan of care for this women? (Select all that apply.) A) Administration of penicillin G at the onset of labor B) Avoidance of scalp electrodes for fetal monitoring C) Refraining from obtaining fetal scalp blood for pH testing D) Administering zidovudine at the onset of labor. E) Electing for the use of forceps-assisted delivery

-Avoidance of scalp electrodes for fetal monitoring -Refraining from obtaining fetal scalp blood for pH testing -Administering zidovudine at the onset of labor

Which position would be most appropriate for the nurse to suggest as a comfort measure to a woman who is in the first stage of labor? (Select all that apply.) A) Walking with partner support B) Straddling with forward leaning over a chair C) Closed knee-chest position D) Rocking back and forth with foot on chair E) Supine with legs raised at a 90-degree angle

-Walking with partner support -Straddling with forward leaning over a chair -Rocking back and forth with foot on chair

Positioning during the first stage of labor includes:

-walking with support from the partner - side-lying with pillows between the knees - leaning forward by straddling a chair, table, or bed or kneeling over a birthing ball -lunging by rocking weight back and forth with a foot up on a chair or birthing ball or an open knee-chest position.

A woman is in her early second trimester of pregnancy. The nurse would instruct the woman to return for a follow-up visit every: A. 2 weeks B. 4 weeks C. 3 weeks D. 1 week

B The recommended follow-up visit schedule is every 4 weeks up to 28 weeks, every 2 weeks from 29 to 36 weeks, and then every week from 37 weeks to birth.

For continuous internal electronic fetal monitoring, four criteria must be met:

1. ruptured membranes 2. cervical dilation of at least 2 cm 3. fetal presenting part low enough to allow placement of the electrode 4. skilled practitioner available to insert the electrode.

The nurse is monitoring a pregnant client admitted to a health care center who is in the latent phase of labor. The nurse demonstrates appropriate nursing care by monitoring the fetal heart rate with the Dopplar at least how often?

30 to 60 min

The woman's temperature is typically assessed every ____ hours during the first stage of labor and every ____ hours after ruptured membranes.

4 2

A 29-week-gestation client is admitted with moderate vaginal discharge. The nurse performs a nitrazine test to determine if the membranes have ruptured. The nitrazine tape remains yellow to olive green, with pH between 5 and 6. What should the nurse do NEXT? A. Prepare the client for birth B. Assess the client's cervical status C. notify the health care provider D. perform leopold's maneuver

5 to 6 pH means acidic environment with presence of vaginal fluid and less blood. notify provider

There is a concern that your client is not obtaining enough folic acid. Which test can be used to evaluate the fetus for potential neural tube defects? A. Alpha-fetoprotein analysis B. Doppler flow study C. Amniocentesis D. Triple-marker screen

A Alpha-fetoprotein is a substance produced by the fetus. AFP enters the maternal circulation by crossing the placenta. If there is a developmental defect, more AFP escapes into amniotic fluid from the fetus. The optimal time for AFP screening is 16 to 18 weeks. The triple marker screens for AFP, hCG, and unconjugated estriol. This screens for neural defects and Down syndrome. The Doppler flow study evaluates the blood flow and amniocentesis evaluates the contents of the amniotic fluid looking for chromosomal defects.

A young couple are concerned that their fetus may be born with sickle cell anemia. You explain that the recessive traits of sickle-cell anemia can be determined by using which test? A.Chorionic villus sampling B. Amniocentesis C. Blood typing D. Percutaneous umbilical blood sampling

A Chorionic villus sampling (CVS) is a procedure for obtaining a sample of the chorionic villi for prenatal evaluation of chromosomal disorders, enzyme deficiencies, fetal gender determination, and to identify sex-linked disorders such as hemophilia, sickle cell anemia, and Tay-Sachs disease. Amniocentesis is used to evaluate for neural tube defects, chromosomal disorders, and inborn errors of metabolism. Blood typing is performed via a blood sample. Percutaneous umbilical blood sampling allows for rapid chromosomal analysis.

The nurse notes persistent early decelerations on the fetal monitoring strip. Which of the following would the nurse do next? A) Continue to monitor the FHR because this pattern is benign. B) Perform a vaginal exam to assess cervical dilation and effacement. C) Stay with the client while reporting the finding to the physician. D) Administer oxygen after turning the client on her left side.

A Early decelerations are not indicative of fetal distress and do not require intervention. Therefore, the nurse would continue to monitor the fetal heart rate pattern. They are most often seen during the active stage of any normal labor, during pushing, crowning, or vacuum extraction. They are thought to be a result of fetal head compression that results in a reflex vagal response with a resultant slowing of the FHR during uterine contractions.

Ans: D The woman is describing afterpains, which are usually stronger during breast-feeding because oxytocin released by the sucking reflex strengthens uterine contractions. Afterpains are associated with uterine involution, but the woman's description strongly correlates with the hormonal events of breast-feeding. All women experience afterpains, but they are more acute in multiparous women secondary to repeated stretching of the uterine muscles.

A client who is breast-feeding her newborn tells the nurse, "I notice that when I feed him, I feel fairly strong contraction-like pain. Labor is over. Why am I having contractions now?" Which response by the nurse would be most appropriate? A) "Your uterus is still shrinking in size; that's why you're feeling this pain." B) "Let me check your vaginal discharge just to make sure everything is fine." C) "Your body is responding to the events of labor, just like after a tough workout." D) "The baby's sucking releases a hormone that causes the uterus to contract."

Ans: B The father's statement reflects transition to mastery because he is making a conscious decision to take control and be at the center of the newborn's life regardless of his preparedness. The expectations stage involves preconceptions about how life will be with a newborn. Reality occurs when fathers realize their expectations are not realistic. Taking-in is a phase of maternal adaptation.

A father of a newborn tells the nurse, "I may not know everything about being a dad, but I'm going to do the best I can for my son." The nurse interprets this as indicating the father is in which stage of adaptation? A) Expectations B) Transition to mastery C) Reality D) Taking-in

Ans: C Attachment is a formation of a relationship between a parent and her/his newborn through a process of physical and emotional interactions. Puerperium refers to the postpartum period. Lactation refers to the process of milk secretion by the breasts. Engrossment refers to the bond that develops between the father and the newborn.

A group of nursing students are reviewing information about maternal and paternal adaptations to the birth of a newborn. The nurse observes the parents interacting with their newborn physically and emotionally. The nurse documents this as which of the following? A) Puerperium B) Lactation C) Attachment D) Engrossment

Ans: B Respirations usually remain within the normal adult range of 16 to 24 breaths per minute. As the abdominal organs resume their nonpregnant position, the diaphragm returns to its usual position. Anatomic changes in the thoracic cavity and rib cage caused by increasing uterine growth resolve quickly. As a result, discomforts such as shortness of breath and rib aches are relieved.

A group of nursing students are reviewing respiratory system adaptations that occur during the postpartum period. The students demonstrate understanding of the information when they identify which of the following as a postpartum adaptation? A) Continued shortness of breath B) Relief of rib aching C) Diaphragmatic elevation D) Decrease in respiratory rate

Ans: D Oxytocin is released from the posterior pituitary to promote milk let-down. Prolactin levels increase at term with a decrease in estrogen and progesterone; estrogen and progesterone levels decrease after the placenta is delivered. Prolactin is released from the anterior pituitary gland and initiates milk production.

A group of students are reviewing the process of breast milk production. The students demonstrate understanding when they identify which hormone as responsible for milk let-down? A) Prolactin B) Estrogen C) Progesterone D) Oxytocin

Ans: C The first task of adjusting to the maternal role is the taking-in phase in which the mother demonstrates dependent behaviors and assumes a passive role in meeting her own basic needs. During the taking-hold phase, the mother becomes preoccupied with the present. During the letting-go phase, the mother reestablishes relationships with others and demonstrates increased responsibility and confidence in caring for the newborn.

A nurse is assessing a postpartum woman's adjustment to her maternal role. Which of the following would the nurse expect to occur first? A) Reestablishing relationships with others B) Demonstrating increasing confidence in care of the newborn C) Assuming a passive role in meeting her own needs D) Becoming preoccupied with the present

Ans: B Engorgement is the process of swelling of the breast tissue as a result of an increase in blood and lymph supply as a precursor to lactation (Figure 15.4). Breast engorgement usually peaks in 3 to 5 days postpartum and usually subsides within the next 24 to 36 hours (Chapman, 2011). Engorgement can occur from infrequent feeding or ineffective emptying of the breasts and typically lasts about 24 hours. Breasts increase in vascularity and swell in response to prolactin 2 to 4 days after birth. If engorged, the breasts will be hard and tender to touch. Involution refers to the process of the uterus returning to its prepregnant state. Mastitis refers to an infection of the breasts. Engrossment refers to the bond that develops between the father and the newborn.

A nurse is making a home visit to a postpartum woman who delivered a healthy newborn 4 days ago. The woman's breasts are swollen, hard, and tender to the touch. The nurse documents this finding as which of the following? A) Involution B) Engorgement C) Mastitis D) Engrossment

Ans: B Lochia serosa is pinkish brown and is expelled 3 to 10 days postpartum. Lochia rubra is a deep-red mixture of mucus, tissue debris, and blood that occurs for the first 3 to 4 days after birth. Lochia alba is creamy white or light brown and consists of leukocytes, decidual tissue, and reduced fluid content and occurs from days 10 to 14 but can last 3 to 6 weeks postpartum.

A nurse is visiting a postpartum woman who delivered a healthy newborn 5 days ago. Which of the following would the nurse expect to find? A) Bright red discharge B) Pinkish brown discharge C) Deep red mucus-like discharge D) Creamy white discharge

The nurse is performing Leopold's maneuvers to determine fetal presentation, position, and lie. Which action would the nurse do first? A) Feel for the fetal buttocks or head while palpating the abdomen. B) Feel for the fetal back and limbs as the hands move laterally on the abdomen. C) Palpate for the presenting part in the area just above the symphysis pubis. D) Determine flexion by pressing downward toward the symphysis pubis.

A. - The first maneuver involves feeling for the buttocks and head. - Next the nurse palpates on which side the fetal back is located. - The third maneuver determines presentation and involves palpating the area just above the symphysis pubis. - The final maneuver determines attitude and involves applying downward pressure in the direction of the symphysis pubis.

Ans: D Clotting factors that increased during pregnancy tend to remain elevated during the early postpartum period. Giving birth stimulates this hypercoagulability state further. As a result, these coagulation factors remain elevated for 2 to 3 weeks postpartum (Silver & Major, 2010). This hypercoagulable state, combined with vessel damage during birth and immobility, places the woman at risk for thromboembolism (blood clots) in the lower extremities and the lungs. Red blood cell production ceases early in the puerperium, which causes mean hemoglobin and hematocrit levels to decrease slightly in the first 24 hours and then rise slowly over the next 2 weeks.

A nurse teaches a postpartum woman about her risk for thromboembolism. Which of the following would the nurse be least likely to include as a factor increasing her risk? A) Increased clotting factors B) Vessel damage C) Immobility D) Increased red blood cell production

Ans: B, C, D Many women have difficulty feeling the sensation to void after giving birth if they received an anesthetic block during labor (which inhibits neural functioning of the bladder) or if they received oxytocin to induce or augment their labor (antidiuretic effect). These women will be at risk for incomplete emptying, bladder distention, difficulty voiding, and urinary retention. In addition, urination may be impeded by perineal lacerations; generalized swelling and bruising of the perineum and tissues surrounding the urinary meatus; hematomas; decreased bladder tone as a result of regional anesthesia; and diminished sensation of bladder pressure as a result of swelling, poor bladder tone, and numbing effects of regional anesthesia used during labor.

A nursing student is preparing a class presentation about changes in the various body systems during the postpartum period and their effects. Which of the following would the student include as influencing a postpartum woman's ability to void? (Select all that apply.) A) Use of an opioid anesthetic during labor B) Generalized swelling of the perineum C) Decreased bladder tone from regional anesthesia D) Use of oxytocin to augment labor E) Need for an episiotomy

Ans: D The external cervical os is no longer shaped like a circle but instead appears as a jagged slit-like opening, often described as a "fish mouth."

A postpartum client comes to the clinic for her 6-week postpartum checkup. When assessing the client's cervix, the nurse would expect the external cervical os to appear: A) Shapeless B) Circular C) Triangular D) Slit-like

Ans: A Factors that inhibit involution include prolonged labor and difficult birth, incomplete expulsion of amniotic membranes and placenta, uterine infection, overdistention of uterine muscles (such as by multiple gestation, hydramnios, or large singleton fetus), full bladder (which displaces the uterus and interferes with contractions), anesthesia (which relaxes uterine muscles), and close childbirth spacing. Factors that facilitate uterine involution include complete expulsion of amniotic membranes and placenta at birth, complication-free labor and birth process, breast-feeding, and early ambulation.

A postpartum client is experiencing subinvolution. When reviewing the woman's labor and birth history, which of the following would the nurse identify as being least significant to this condition? A) Early ambulation B) Prolonged labor C) Large fetus D) Use of anesthetics

Ans: B For the nonlactating woman, menstruation resumes 7 to 9 weeks after giving birth, with the first cycle being anovulatory. For the lactating woman, menses can return anytime from 2 to 18 months after childbirth.

A postpartum client who is bottle feeding her newborn asks, "When should my period return?" Which response by the nurse would be most appropriate? A) "It's difficult to say, but it will probably return in about 2 to 3 weeks." B) "It varies, but you can estimate it returning in about 7 to 9 weeks." C) "You won't have to worry about it returning for at least 3 months." D) "You don't have to worry about that now. It'll be quite a while."

Ans: B Separation of the rectus abdominis muscles, called diastasis recti, is more common in women who have poor abdominal muscle tone before pregnancy. After birth, muscle tone is diminished and the abdominal muscles are soft and flabby. Specific exercises are necessary to help the woman regain muscle tone. Fortunately, diastasis responds well to exercise, and abdominal muscle tone can be improved. Stretch marks (striae gravidarum) fade to silvery lines. The darkened pigmentation of the abdomen (linea nigra), face (melasma), and nipples gradually fades. Parous women will note a permanent increase in shoe size.

A postpartum woman who has experienced diastasis recti asks the nurse about what to expect related to this condition. Which response by the nurse would be most appropriate? A) "You'll notice that this will fade to silvery lines." B) "Exercise will help to improve the muscles." C) "Expect the color to lighten somewhat." D) "You'll notice that your shoe size will increase."

Ans: C During the first few days after birth, the uterus typically descends downward from the level of the umbilicus at a rate of 1 cm (1 fingerbreadth) per day so that by day 2, it is about 2 fingerbreadths below the umbilicus.

A primipara client gave birth vaginally to a healthy newborn girl 48 hours ago. The nurse palpates the client's fundus, expecting it to be at which location? A) Two fingerbreadths above the umbilicus B) At the level of the umbilicus C) Two fingerbreadths below the umbilicus D) Four fingerbreadths below the umbilicus

Ans: A Although hCG, hPL, and progesterone decline rapidly after birth, decreased estrogen levels are associated with breast engorgement and with the diuresis of excess extracellular fluid accumulated during pregnancy.

A woman who delivered a healthy newborn several hours ago asks the nurse, "Why am I perspiring so much?" The nurse integrates knowledge that a decrease in which hormone plays a role in this occurrence? A) Estrogen B) hCG C) hPL D) Progesterone

Which of the following is a priority when caring for a woman during the fourth stage of labor? A) Assessing the uterine fundus B) Offering fluids as indicated C) Encouraging the woman to void D) Assisting with perineal care

A. During the fourth stage of labor, a priority is to assess the woman's fundus to prevent postpartum hemorrhage.

Ans: B Postpartum diuresis occurs as a result of several mechanisms: the large amounts of IV fluids given during labor, a decreasing antidiuretic effect of oxytocin as its level declines, the buildup and retention of extra fluids during pregnancy, and a decreasing production of aldosterone—the hormone that decreases sodium retention and increases urine production. All these factors contribute to rapid filling of the bladder within 12 hours of birth. Diuresis begins within 12 hours after childbirth and continues throughout the first week postpartum. Rapid bladder filling, possible infection, or effects of anesthesia are not involved.

A woman who gave birth 24 hours ago tells the nurse, "I've been urinating so much over the past several hours." Which response by the nurse would be most appropriate? A) "You must have an infection, so let me get a urine specimen." B) "Your body is undergoing many changes that cause your bladder to fill quickly." C) "Your uterus is not contracting as quickly as it should." D) "The anesthesia that you received is wearing off and your bladder is working again."

A nurse palpates a woman's fundus to determine contraction intensity. Which of the following would be most appropriate for the nurse to use for palpation? A) Finger pads B) Palm of the hand C) Finger tips D) Back of the hand

A. To palpate the fundus for contraction intensity, the nurse would place the pads of the fingers on the fundus and describe how it feels.

Your client is in her second trimester and the health care provider has recommended she undergo an amniocentesis. You explain the procedure is used to diagnose which of the following? (Select all that apply.) A. Inborn errors of metabolism B. Neural tube defects C. Rh incompatibility D. Chromosomal abnormalities E. HIV

A,B,D Amniocentesis is performed in the second trimester, usually between 15 and 18 weeks gestation. Over 40 different chromosomal abnormalities, inborn errors of metabolism, and neural tube defects can be diagnosed with amniocentesis. It can replace a genetic probability with a diagnostic certainty, allowing the woman and her partner to make an informed decision about their options. Rh incompatibility and HIV status are both evaluated by blood tests.

The nurse is preparing a teaching plan for a pregnant woman about the signs and symptoms to be reported immediately to her health care provider. Which of the following would the nurse include? (Select all that apply.) A. Sudden leakage of fluid during the second trimester B. Headache with visual changes in the third trimester C. Nausea with vomiting during the first trimester D. Lower abdominal pain with shoulder pain in the first trimester E. Urinary frequency in the third trimester

A,B,D Danger signs and symptoms that need to be reported immediately include headache with visual changes and sudden leakage of fluid in the second trimester, and lower abdominal pain accompanied by shoulder pain in the first trimester. Urinary frequency in the third trimester, nausea and vomiting during the first trimester, and backache during the second trimester are common discomforts of pregnancy.

A pregnant woman in her second trimester tells the nurse, "I've been passing a lot of gas and feel bloated." Which of the following suggestions would be helpful for the woman? A) "Watch how much beans and onions you eat." B) "Limit the amount of fluid you drink with meals" C) "Try exercising a little more." D) "Some say that eating mints can help." E) "Cut down on your intake of cheeses."

A,C,D

Your client is complaining at her prenatal visit that she cannot find any shoes that are comfortable. Assessment of her legs reveals dependent edema. You suggest she attempt to do the following to help reduce the edema: (Select all that apply.) A. Elevate feet and legs when sitting or lying. B. When lying down, lie on the right side. C. Avoid foods high in sodium, sugar, and fats. D. Drink 6 to 8 glasses of water each day. E. Wear knee-high support stockings.

A,C,D Dependent edema is usually the result of pressure put on the veins preventing adequate blood flow to return to the heart. Appropriate suggestions to reduce dependent edema include elevating feet and legs when sitting or lying down; avoiding foods that are high in sodium, sugar, and fats; drinking at least 6 to 8 glasses of water per day; avoid wearing knee-high stockings; and lying on the left side to keep the gravid uterus off the vena cava to return blood to the heart.

A pregnant woman who is 26 weeks pregnant arrives for a follow-up visit. Which of the following assessments, in addition to measuring fundal height and fetal heart rate, would the nurse expect to complete? (Select all that apply.) A. Blood glucose level B. Edema C. Blood pressure D. Weight E. Urine testing

A,C,D,E Up to 28 weeks' gestation, follow-up visits involve assessment of the client's blood pressure and weight, urine testing for protein and glucose, along with fundal height and fetal heart rate. Between weeks 24 and 28, a blood glucose level is obtained. Assessment for edema is typically done between 29 and 36 weeks' gestation.

A woman in labor received an opioid close to the time of birth. The nurse would assess the newborn for which of the following? A) Respiratory depression B) Urinary retention C) Abdominal distention D) Hyperreflexia

A.

A woman has just entered the second stage of labor. The nurse would focus care on which of the following? A) Encouraging the woman to push when she has a strong desire to do so B) Alleviating perineal discomfort with the application of ice packs C) Palpating the woman's fundus for position and firmness D) Completing the identification process of the newborn with the mother

A. During the second stage of labor, nursing interventions focus on motivating the woman, encouraging her to put all her efforts toward pushing.

A nurse is describing the different types of regional analgesia and anesthesia for labor to a group of pregnant women. Which statement by the group indicates that the teaching was successful? A) "We can get up and walk around after receiving combined spinal-epidural analgesia." B) "Higher anesthetic doses are needed for patient-controlled epidural analgesia. C) "A pudendal nerve block is highly effective for pain relief in the first stage of labor." D) "Local infiltration using lidocaine is an appropriate method for controlling contraction pain."

A. When compared with traditional epidural or spinal analgesia, which often keeps the woman lying in bed, combined spinal-epidural analgesia allows the woman to ambulate ("walking epidural").

A client's membranes spontaneously ruptured, as evidenced by a gush of clear fluid with a contraction. Which of the following would the nurse do next? A) Check the fetal heart rate. B) Perform a vaginal exam. C) Notify the physician immediately. D) Change the linen saver pad.

A. When membranes rupture, the PRIORITY focus is on assessing fetal heart rate first to identify a deceleration, which might indicate cord compression secondary to cord prolapse.

Ans: D Involution involves three retrogressive Process: contraction of muscle fibers to reduce those previously stretched during pregnancy; catabolism, which reduces enlarged myometrial cells; and regeneration of uterine epithelium from the lower layer of the decidua after the upper layers have been sloughed off and shed during lochial discharge. Vasodilation is not involved.

After teaching a group of nursing students about the process of involution, the instructor determines that additional teaching is needed when the students identify which of the following as being involved? A) Catabolism B) Muscle fiber contraction C) Epithelial regeneration D) Vasodilation

Which of the following would be most appropriate for the nurse to suggest about pushing to a woman in the second stage of labor? A) "Lying flat with your head elevated on two pillows makes pushing easier." B) "Choose whatever method you feel most comfortable with for pushing." C) " Let me help you decide when it is time to start pushing." D) "Bear down like you're having a bowel movement with every contraction."

Ans. B "Choose whatever method you feel most comfortable with for pushing." * The role of the nurse should be to support the woman in her choice of pushing method and to encourage confidence in her maternal instinct of when and how to push.

When assessing a woman in her first trimester, which emotional response would the nurse most likely expect to find? A) Ambivalence B) Introversion C) Acceptance D) Emotional lability

Ans: A During the first trimester, the pregnant woman commonly experiences ambivalence, with conflicting feelings at the same time. Introversion heightens during the first and third trimesters when the woman's focus is on behaviors that will ensure a safe and healthy pregnancy outcome. Acceptance usually occurs during the second trimester. Emotional lability (mood swings) is characteristic throughout a woman's pregnancy.

19. The nurse is reviewing the monitoring strip of a woman in labor who is experiencing a contraction. The nurse notes the time the contraction takes from its onset to reach its highest intensity. The nurse interprets this time as which of the following? A) Increment B) Acme C) Peak D) Decrement

Ans: A Feedback: Each contraction has three phases: increment or the buildup of the contraction; acme or the peak or highest intensity; and the decrement or relaxation of the uterine muscle fibers. The time from the onset to the highest intensity corresponds to the increment.

13. Assessment of a fetus identifies the buttocks as the presenting part, with the legs extended upward. The nurse identifies this as which type of breech presentation? A) Frank B) Full C) Complete D) Footling

Ans: A Feedback: In a frank breech, the buttocks present first, with both legs extended up toward the face. In a full or complete breech, the fetus sits cross-legged above the cervix. In a footling breech, one or both legs are presenting.

24. During a follow-up prenatal visit, a pregnant woman asks the nurse, "How long do you think I will be in labor?" Which response by the nurse would be most appropriate? A) "It's difficult to predict how your labor will progress, but we'll be there for you the entire time." B) "Since this is your first pregnancy, you can estimate it will be about 10 hours." C) "It will depend on how big the baby is when you go into labor." D) "Time isn't important; your health and the baby's health are key."

Ans: A Feedback: It is difficult to predict how a labor will progress and therefore equally difficult to determine how long a woman's labor will last. There is no way to estimate the likely strength and frequency of uterine contractions, the extent to which the cervix will soften and dilate, and how much the fetal head will mold to fit the birth canal. We cannot know beforehand whether the complex fetal rotations needed for an efficient labor will take place properly. All of these factors are unknowns when a woman starts labor. Telling the woman an approximate time would be inappropriate because there is no way to determine the length of labor. It is highly individualized. Although fetal size and maternal and fetal health are important considerations, these responses do not address the woman's concern.

2. Which of the following would indicate to the nurse that the placenta is separating? A) Uterus becomes globular B) Fetal head is at vaginal opening C) Umbilical cord shortens D) Mucous plug is expelled

Ans: A Feedback: Placental separation is indicated by the uterus changing shape to globular and upward rising of the uterus. Additional signs include a sudden trickle of blood from the vaginal opening, and lengthening (not shortening) of the umbilical cord. The fetal head at the vaginal opening is termed crowning and occurs before birth of the head. Expulsion of the mucous plug is a premonitory sign of labor.

25. A nurse is describing how the fetus moves through the birth canal. Which of the following would the nurse identify as being most important in allowing the fetal head to move through the pelvis? A) Sutures B) Fontanelles C) Frontal bones D) Biparietal diameter

Ans: A Feedback: Sutures are important because they allow the cranial bones to overlap in order for the head to adjust in shape (elongate) when pressure is exerted on it by uterine contractions or the maternal bony pelvis. Fontanelles are the intersections formed by the sutures. The frontal bones, along with the parietal and occipital bones are bones of the cranium that are soft and pliable. The biparietal diameter is an important diameter that can affect the birth process.

7. The fetus of a nulliparous woman is in a shoulder presentation. The nurse would most likely prepare the client for which type of birth? A) Cesarean B) Vaginal C) Forceps-assisted D) Vacuum extraction

Ans: A Feedback: The fetus is in a transverse lie with the shoulder as the presenting part, necessitating a cesarean birth. Vaginal birth, forceps-assisted, and vacuum extraction births are not appropriate.

8. Assessment of a woman in labor reveals cervical dilation of 3 cm, cervical effacement of 30%, and contractions occurring every 7 to 8 minutes, lasting about 40 seconds. The nurse determines that this client is in: A) Latent phase of the first stage B) Active phase of the first stage C) Transition phase of the first stage D) Perineal phase of the second stage

Ans: A Feedback: The latent phase of the first stage of labor involves cervical dilation of 0 to 3 cm, cervical effacement of 0% to 40%, and contractions every 5 to 10 minutes lasting 30 to 45 seconds. The active phase is characterized by cervical dilation of 4 to 7 cm, effacement of 40% to 80%, and contractions occurring every 2 to 5 minutes lasting 45 to 60 seconds. The transition phase is characterized by cervical dilation of 8 to 10 cm, effacement of 80% to 100%, and contractions occurring every 1 to 2 minutes lasting 60 to 90 seconds. The perineal phase of the second stage occurs with complete cervical dilation and effacement, contractions occurring every 2 to 3 minutes and lasting 60 to 90 seconds, and a tremendous urge to push by the mother.

5. A woman is in the first stage of labor. The nurse would encourage her to assume which position to facilitate the progress of labor? A) Supine B) Lithotomy C) Upright D) Knee-chest

Ans: C Feedback: The use of any upright position helps to reduce the length of labor. Research validates that nonmoving back-lying positions such as supine and lithotomy positions during labor are not healthy. The knee-chest position would assist in rotating the fetus in a posterior position.

15. After teaching a group of students about the factors affecting the labor process, the instructor determines that the teaching was successful when the group identifies which of the following as a component of the true pelvis? (Select all that apply.) A) Pelvic inlet B) Cervix C) Mid pelvis D) Pelvic outlet E) Vagina F) Pelvic floor muscles

Ans: A, C, D Feedback: The true pelvis is made up of three planes: the pelvic inlet, mid pelvis, and pelvic outlet. The cervix, vagina, and pelvic floor muscles are the soft tissues of the passageway.

23. A nurse is preparing a presentation for a group of pregnant women about the labor experience. Which of the following would the nurse most likely include when discussing measures to promote coping for a positive labor experience? (Select all that apply.) A) Presence of a support partner B) View of birth as a stressor C) Low anxiety level D) Fear of loss of control E) Participation in a pregnancy exercise program

Ans: A, C, E Feedback: Numerous factors can affect a woman's coping ability during labor and birth. Having the presence and support of a valued partner during labor, engaging in exercise during pregnancy, viewing the birthing experience as a meaningful rather than stressful event, and a low anxiety level can promote a woman's ability to cope. Excessive anxiety may interfere with the labor progress, and fear of labor and loss of control may enhance pain perception, increasing the fear.

18. A nurse is preparing a class for pregnant women about labor and birth. When describing the typical movements that the fetus goes through as it travels through the passageway, which of the following would the nurse most likely include? (Select all that apply.) A) Internal rotation B) Abduction C) Descent D) Pronation E) Flexion

Ans: A, C, E Feedback: The positional changes that occur as the fetus moves through the passageway are called the cardinal movements of labor and include engagement, descent, flexion, internal rotation, extension, external rotation, and expulsion. The fetus does not undergo abduction or pronation.

3. When assessing cervical effacement of a client in labor, the nurse assesses which of the following characteristics? A) Extent of opening to its widest diameter B) Degree of thinning C) Passage of the mucous plug D) Fetal presenting part

Ans: B Feedback: Effacement refers to the degree of thinning of the cervix. Cervical dilation refers to the extent of opening at the widest diameter. Passage of the mucous plug occurs with bloody show is a premonitory sign of labor. The fetal presenting part is determined by vaginal examination and is commonly the head (cephalic), pelvis (breech), or shoulder.

14. A woman in her third trimester comes to the clinic for a prenatal visit. During assessment the woman reports that her breathing has become much easier in the last week but she has noticed increased pelvic pressure, cramping, and lower back pain. The nurse determines that which of the following has most likely occurred? A) Cervical dilation B) Lightening C) Bloody show D) Braxton-Hicks contractions

Ans: B Feedback: Lightening occurs when the fetal presenting part begins to descend into the maternal pelvis. The uterus lowers and moves into the maternal pelvis. The shape of the abdomen changes as a result of the change in the uterus. The woman usually notes that her breathing is much easier. However, she may complain of increased pelvic pressure, cramping, and lower back pain. Although cervical dilation also may be occurring, it does not account for the woman's complaints. Bloody show refers to passage of the mucous plug that fills the cervical canal during pregnancy. It occurs with the onset of labor. Braxton-Hicks contractions increase in strength and frequency and aid in moving the cervix from a posterior position to an anterior position. They also help in ripening and softening the cervix.

4. A woman calls the health care facility stating that she is in labor. The nurse would urge the client to come to the facility if the client reports which of the following? A) Increased energy level with alternating strong and weak contractions B) Moderately strong contractions every 4 minutes, lasting about 1 minute C) Contractions noted in the front of abdomen that stop when she walks D) Pink-tinged vaginal secretions and irregular contractions lasting about 30 seconds

Ans: B Feedback: Moderately strong regular contractions 60 seconds in duration indicate that the client is probably in the active phase of the first stage of labor. Alternating strong and weak contractions, contractions in the front of the abdomen that change with activity, and pink-tinged secretions with irregular contractions suggest false labor.

11. After teaching a group of students about the maternal bony pelvis, which statement by the group indicates that the teaching was successful? A) The bony pelvis plays a lesser role during labor than soft tissue. B) The pelvic outlet is associated with the true pelvis. C) The false pelvis lies below the imaginary linea terminalis. D) The false pelvis is the passageway through which the fetus travels.

Ans: B Feedback: The maternal bony pelvis consists of the true and false portions. The true pelvis is made up of three planes—the inlet, the mid pelvis, and the outlet. The bony pelvis is the more important part of the passageway because it is relatively unyielding. The false pelvis lies above the imaginary linea terminalis. The true pelvis is the bony passageway through which the fetus must travel.

9. A client is admitted to the labor and birthing suite in early labor. On review of her medical record, the nurse determines that the client's pelvic shape as identified in the antepartal progress notes is the most favorable one for a vaginal delivery. Which pelvic shape would the nurse have noted? A) Platypelloid B) Gynecoid C) Android D) Anthropoid

Ans: B Feedback: The most favorable pelvic shape for vaginal delivery is the gynecoid shape. The anthropoid pelvis is favorable for vaginal birth but it is not the most favorable shape. The android pelvis is not considered favorable for a vaginal birth because descent of the fetal head is slow and failure of the fetus to rotate is common. Women with a platypelloid pelvis usually require cesarean birth.

21. When describing the stages of labor to a pregnant woman, which of the following would the nurse identify as the major change occurring during the first stage? A) Regular contractions B) Cervical dilation C) Fetal movement through the birth canal D) Placental separation

Ans: B Feedback: The primary change occurring during the first stage of labor is progressive cervical dilation. Contractions occur during the first and second stages of labor. Fetal movement through the birth canal is the major change during the second stage of labor. Placental separation occurs during the third stage of labor.

10. A woman telephones her health care provider and reports that her "water just broke." Which suggestion by the nurse would be most appropriate? A) "Call us back when you start having contractions." B) "Come to the clinic or emergency department for an evaluation." C) "Drink 3 to 4 glasses of water and lie down." D) "Come in as soon as you feel the urge to push."

Ans: B Feedback: When the amniotic sac ruptures, the barrier to infection is gone and there is the danger of cord prolapse if engagement has not occurred. Therefore, the nurse should suggest that the woman come in for an evaluation. Calling back when contractions start, drinking water, and lying down are inappropriate because of the increased risk for infection and cord prolapse. Telling the client to wait until she feels the urge to push is inappropriate because this occurs during the second stage of labor.

1. A woman in her 40th week of pregnancy calls the nurse at the clinic and says she's not sure whether she is in true or false labor. Which statement by the client would lead the nurse to suspect that the woman is experiencing false labor? A) "I'm feeling contractions mostly in my back." B) "My contractions are about 6 minutes apart and regular." C) "The contractions slow down when I walk around." D) "If I try to talk to my partner during a contraction, I can't."

Ans: C Feedback: False labor is characterized by contractions that are irregular and weak, often slowing down with walking or a position change. True labor contractions begin in the back and radiate around toward the front of the abdomen. They are regular and become stronger over time; the woman may find it extremely difficult if not impossible to have a conversation during a contraction.

16. A nurse is documenting fetal lie of a woman in labor. Which term would the nurse most likely use? A) Flexion B) Extension C) Longitudinal D) Cephalic

Ans: C Feedback: Fetal lie refers to the relationships of the long axis (spine) of the fetus to the long axis (spine) of the mother. There are two primary lies: longitudinal and transverse. Flexion and extension are terms used to describe fetal attitude. Cephalic is a term used to describe fetal presentation.

26. Assessment of a pregnant woman reveals that the presenting part of the fetus is at the level of the maternal ischial spines. The nurse documents this as which station? A) -2 B) -1 C) 0 D) +1 .

Ans: C Feedback: Station refers to the relationship of the presenting part to the level of the maternal pelvic ischial spines. Fetal station is measured in centimeters and is referred to as a minus or plus, depending on its location above or below the ischial spines. Zero (0) station is designated when the presenting part is at the level of the maternal ischial spines. When the presenting part is above the ischial spines, the distance is recorded as minus stations. When the presenting part is below the ischial spines, the distance is recorded as plus stations

17. The nurse is reviewing the medical record of a woman in labor and notes that the fetal position is documented as LSA. The nurse interprets this information as indicating which of the following is the presenting part? A) Occiput B) Face C) Buttocks D) Shoulder

Ans: C Feedback: The second letter denotes the presenting part which in this case is "S" or the sacrum or buttocks. The letter "O" would denote the occiput or vertex presentation. The letter "M" would denote the mentum (chin) or face presentation. The letter "A" would denote the acromion or shoulder presentation.

6. A client has not received any medication during her labor. She is having frequent contractions every 1 to 2 minutes and has become irritable with her coach and no longer will allow the nurse to palpate her fundus during contractions. Her cervix is 8 cm dilated and 90% effaced. The nurse interprets these findings as indicating: A) Latent phase of the first stage of labor B) Active phase of the first stage of labor C) Transition phase of the first stage of labor D) Pelvic phase of the second stage of labor

Ans: C Feedback: The transition phase is characterized by cervical dilation of 8 to 10 cm, effacement of 80% to 100%, contractions that are strong, painful, and frequent (every 1 to 2 minutes) and last 60 to 90 seconds, and irritability, apprehension, and feelings of loss of control. The latent phase is characterized by mild contractions every 5 to 10 minutes, cervical dilation of 0 to 3 cm and effacement of 0% to 40%, and excitement and frequent talking by the mother. The active phase is characterized by moderate to strong contractions every 2 to 5 minutes, cervical dilation of 4 to 7 cm and effacement of 40% to 80%, with the mother becoming intense and inwardly focused. The pelvic phase of the second stage of labor is characterized by complete cervical dilation and effacement, with strong contractions every 2 to 3 minutes; the mother focuses on pushing.

20. A nurse is assessing a woman in labor. Which finding would the nurse identify as a cause for concern during a contraction? A) Heart rate increase from 76 bpm to 90 bpm B) Blood pressure rise from 110/60 mm Hg to 120/74 C) White blood cell count of 12,000 cells/mm3 D) Respiratory rate of 10 breaths /minute

Ans: D Feedback: During labor, the mother experiences various physiologic responses including an increase in heart rate by 10 to 20 bpm, a rise in blood pressure by up to 35 mm Hg during a contraction, an increase in white blood cell count to 25,000 to 30,000 cells/mm3, perhaps as a result of tissue trauma, and an increase in respiratory rate with greater oxygen consumption due to the increase in metabolism. A drop in respiratory rate would be a cause for concern.

22. A nurse is caring for several women in labor. The nurse determines that which woman is in the transition phase of labor? A) Contractions every 5 minutes, cervical dilation 3 cm B) Contractions every 3 minutes, cervical dilation 5 cm C) Contractions every 21/2 minutes, cervical dilation 7 cm D) Contractions every 1 minute, cervical dilation 9 cm

Ans: D Feedback: The transition phase is characterized by strong contractions occurring every 1 to 2 minutes and cervical dilation from 8 to 10 cm. Contractions every 5 minutes with cervical dilation of 3 cm is typical of the latent phase. Contractions every 3 minutes with cervical dilation of 5 cm and contractions every 21/2 minutes with cervical dilation of 7 cm suggest the active phase of labor.

12. A fetus is assessed at 2 cm above the ischial spines. The nurse would document fetal station as: A) +4 B) +2 C) 0 D) -2

Ans: D Feedback: When the presenting part is above the ischial spines, it is noted as a negative station. Since the measurement is 2 cm, the station would be -2. A 0 station indicates that the fetal presenting part is at the level of the ischial spines. Positive stations indicate that the presenting part is below the level of the ischial spines.

A couple comes to the clinic for preconception counseling and care. As part of the visit, the nurse teaches the couple about fertilization and initial development, stating that the zygote formed by the union of the ovum and sperm consists of how many chromosomes? A) 22 B) 23 C) 44 D) 46

Ans: D Feedback: With fertilization, the ovum, containing 23 chromosomes, and the sperm, containing 23 chromosomes, join, forming a zygote with a diploid number or 46 chromosomes.

A client's last menstrual period was April 11. Using Nagele's rule, her expected date of birth (EDB. would be: A) January 4 B) January 18 C) January 25 D) February 24

B

A client's maternal serum alpha-fetoprotein (MSAFP. level was unusually elevated at 17 weeks. The nurse suspects which of the following? A) Fetal hypoxia B) Open spinal defects C) Down syndrome D) Maternal hypertension

B

A nurse is reviewing the medical record of a pregnant woman and notes that she is gravid II. The nurse interprets this to indicate the number of: A) Deliveries B) Pregnancies C) Spontaneous abortions D) Pre-term births

B

After teaching a group of students about the different perinatal education methods, the instructor determines that the teaching was successful when the students identify which of the following as the Bradley method? A) Psychoprophylactic method B) Partner-coached method C) Natural childbirth method D) Mind prevention method

B

During a routine prenatal visit, a client, 36 weeks pregnant, states she has difficulty breathing and feels like her pulse rate is really fast. The nurse finds her pulse to be 100 beats per minute (increased from baseline readings of 70 to 74 beats per minute. and irregular, with bilateral crackles in the lower lung bases. Which nursing diagnosis would be the priority for this client? A) Ineffective tissue perfusion related to supine hypotensive syndrome B) Impaired gas exchange related to pulmonary congestion C) Activity intolerance related to increased metabolic requirements D) Anxiety related to fear of pregnancy outcome

B

A woman is 20 weeks pregnant. The nurse would expect to palpate the fundus at which of the following locations? A) Just below the ensiform cartilage B) At the umbilicus C) Between the symphysis and umbilicus D) Symphysis pubis

B At 20 weeks' gestation, the fundus can be palpated at the umbilicus. A fundus of 12 weeks' gestation is palpated at the symphysis pubis. At 16 weeks' gestation, the fundus is midway between the symphysis pubis and umbilicus. At 36 weeks' gestation, the fundus can be palpated just below the ensiform cartilage.

When describing the role of a doula to a group of pregnant women, which of the following would the nurse include? A)The doula is capable of handling high-risk births and emergencies. B) The doula primarily focuses on providing continuous labor support. C)The doula can perform any necessary clinical procedures. D) The doula is a professionally trained nurse hired to provide physical and emotional support.

B Doulas provide the woman with continuous support throughout labor. The doula is a laywoman trained to provide women and families with encouragement, emotional and physical support, and information through late pregnancy, labor, and birth. A doula does not perform any clinical procedures and is not trained to handle high-risk births and emergencies.

After teaching the pregnant woman about ways to minimize flatulence and bloating during pregnancy, which statement indicates the need for additional teaching? A) "I'll stay away from foods like cabbage and brussels sprouts." B)"I'll switch to chewing gum instead of using mints." C)"I'll increase my time spent on walking each day." D) "I'll try to drink more fluids to help move things along."

B Eating mints can help reduce flatulence; chewing gum increases the amount of air that is swallowed, increasing gas build-up. Increasing fluid intake helps to reduce flatus. Gas-forming foods such as beans, cabbage, and onions should be avoided. Increasing physical exercise, such as walking, aids in reducing

After teaching the pregnant woman about ways to minimize flatulence and bloating during pregnancy, which statement indicates the need for additional teaching? A. "I'll stay away from foods like cabbage and brussels sprouts." B."I'll switch to chewing gum instead of using mints." C. "I'll increase my time spent on walking each day." D. "I'll try to drink more fluids to help move things along."

B Eating mints can help reduce flatulence; chewing gum increases the amount of air that is swallowed, increasing gas build-up. Increasing fluid intake helps to reduce flatus. Gas-forming foods such as beans, cabbage, and onions should be avoided. Increasing physical exercise, such as walking, aids in reducing flatus.

The initial prenatal visit should be a complete assessment of the client's past and present history. Part of this assessment should include the immunization history. You inform the client to avoid which of the following types of vaccines while she is pregnant? A. Live virus vaccine B. Bacterial vaccine C. Toxoid vaccine D. Inactivated virus vaccine

B Routine immunizations are not usually indicated during pregnancy. However, no evidence exists of risk from vaccinating pregnant women with inactivated virus or bacterial vaccines or toxoids. A number of other vaccines have not been adequately studied. Advise pregnant women to avoid live virus vaccines (MMR and varicella) and to avoid becoming pregnant within one month of having received one of these vaccines because of the theoretical risk of transmission to the fetus.

The diagonal conjugate of a pregnant woman's pelvis is measured. Which measurement would suggest a potential problem? A) 12.5cm B) 12.0 C) 13.5cm D) 13.0

B The diagonal conjugate, usually 12.5 cm or greater, indicates the anteroposterior diameter of the pelvic inlet. The diagonal conjugate is the most useful measurement for estimating pelvic size because a misfit with the fetal head occurs if it is too small.

At the first prenatal visit, the client reports her LMP was November 16, 2011. You determine the estimated date of birth (EDB) to be: A. September 1, 2012 B. August 23, 2012 C. August 3, 2012 D. August 13, 2012

B There are several methods to determine the EDB. Nagele's rule can be used, which involves subtracting 3 months and then adding 7 days to the first day of the LMP. Then correct the year by adding 1 where necessary. Another method is to add 7 days and then add 9 months and add 1 to the year where needed. Thus the client reports her LMP was Nov 16, 2011, subtract 3 months (August), add 7 days (23), and add 1 year (2012). This client's EDB is August 23, 2012.

After describing continuous internal electronic fetal monitoring to a laboring woman and her partner, which of the following would indicate the need for additional teaching? A) "This type of monitoring is the most accurate method for our baby." B) "Unfortunately, I'm going to have to stay quite still in bed while it is in place." C) "This type of monitoring can only be used after my membranes rupture." D) "You'll be inserting a special electrode into my baby's scalp."

B. With continuous internal electronic monitoring, maternal position changes and movement do not interfere with the quality of the tracing. *Continuous internal monitoring is considered the most accurate method, but it can be used only if certain criteria are met, such as rupture of membranes.

A woman in labor is to receive continuous internal electronic fetal monitoring. The nurse reviews the woman's medical record to ensure which of the following as being required? A) Intact membranes B) Cervical dilation of 2 cm or more C) Floating presenting fetal part D) A neonatologist to insert the electrode

B. For continuous internal electronic fetal monitoring, four criteria must be met: 1. ruptured membranes 2. cervical dilation of at least 2 cm 3. fetal presenting part low enough to allow placement of the electrode 4. skilled practitioner available to insert the electrode.

A nurse is reviewing the fetal heart rate pattern and observes abrupt decreases in FHR below the baseline, appearing as a U-shape. The nurse interprets these changes as reflecting which of the following? A) Early decelerations B) Variable decelerations C) Prolonged decelerations D) Late decelerations

B. Variable decelerations present as visually apparent abrupt decreases in FHR below baseline and have an unpredictable shape on the FHR baseline, possibly demonstrating no consistent relationship to uterine contractions. The shape of variable decelerations may be U, V, or W, or they may not resemble other patterns.

A nurse is assessing a woman after birth and notes a second-degree laceration. The nurse interprets this as indicating that the tear extends through which of the following? A) Skin B) Muscles of perineal body C) Anal sphincter D) Anterior rectal wall

B. Muscle of perineal body

A client who is 4 months pregnant is at the prenatal clinic for her initial visit. Her history reveals she has 7-year-old twins who were born at 34 weeks gestation, a 2-year old son born at 39 weeks gestation, and a spontaneous abortion 1 year ago at 6 weeks gestation. Using the GTPAL method, the nurse would document her obstetric history as: A) 3 2 1 0 3 B) 3 1 2 2 3 C) 4 1 1 1 3 D) 4 2 1 3 1

C

The use of real-time ultrasonography allows the health care provider to obtain what type of information about the fetus? A. Chromosomal abnormalities B. The size and shape of placenta C. Biophysical profile D. The effectiveness of neural tube defect treatment

C A biophysical profile uses real-time ultrasound to allow assessment of various parameters of fetal well-being. This may include fetal movements, fetal tone, and fetal breathing, as well as assessment of amniotic fluid volume with or without assessment of fetal heart rate. Chromosomal abnormalities are detected via amniocentesis. Neural tube defect treatment is not evaluated via biophysical profile, and although the placenta may be observed, it is not the focus of this procedure.

A 41-year-old pregnant woman and her husband are anxiously awaiting the results of various blood tests to evaluate the fetus for potential Down syndrome, neural tube defects, and spina bifida. Client education should include which of the following? A. Treatment can be started once the test results are back. B. The blood tests are definitive. C. Further testing will be required to confirm any diagnosis. D. A second set of screening tests can be obtained to confirm results.

C Nursing management related to marker screening tests consists primarily of providing education about the tests. Remind the couple that a definitive diagnosis is not made without further tests such as an amniocentesis. The blood tests are not definitive but only strongly suggest the possibility of a defect. For some conditions there are no treatments. The couple may request a second set, but the health care provider will probably suggest proceeding with the more definitive methods to confirm the diagnosis.

In preparing for a preconception class, it will be important to include a discussion of potential risk factors. Which of the following would be included? A. Importance of taking adequate vitamin and mineral supplements B. The importance of healthy lifestyle C. The use of OTC drugs with teratogens D. Family history of pregnancy complications

C Risk factors for adverse pregnancy have been demonstrated by statistics gathered for smoking during pregnancy, consuming alcohol during pregnancy, not taking adequate folic acid supplements during pregnancy, being obese, taking prescription or OTC drugs that are known teratogens, and having a preexisting condition that can negatively affect pregnancy if unmanaged.

You are leading a discussion among couples who are thinking about getting pregnant. As the nurse you stress that preconception counseling helps to identify risks and encourages modification by the couple before conception. You consider the discussion successful when the couples realize that the greatest risk to the embryo is: A.Between 17 and 56 hours after conception B. Between 4 and 17 hours after conception C. Between 4 and 17 days after conception D. Between 17 and 56 days after conception

C The period of greatest environmental sensitivity and consequent risk for the developing embryo is between days 17 and 56 after conception. The first prenatal visit, which is usually a month or later after a missed menstrual period, may occur too late to affect reproductive outcomes associated with abnormal organogenesis secondary to poor lifestyle choices.

A pregnant woman comes to the clinic for a visit. This is her third pregnancy. She had a miscarriage at 12 weeks and gave birth to a son, now 3 years old, at 32 weeks. Using the GTPAL system, the nurse would document this woman's obstetric history as: A. 31021 B. 21212 C. 30111 D. 20111

C The woman's obstetric history would be documented as 30111, G (gravida) = 3 (current pregnancy), T (term pregnancies) = 0, P (number of preterm pregnancies) = 1, A (number of pregnancies ending before 20 weeks viability) = 1, and L (number of living children) = 1.

Your client is anxious to have an ultrasound at each visit. You explain that it isn't necessary and schedule a second ultrasound to be performed when she is about: A. 18 to 20 weeks pregnant B. 15 to 17 weeks pregnant C. 21 to 23 weeks pregnant D. 24 to 26 weeks pregnant

C There are no hard-and-fast rules as to how many ultrasounds a woman should have during her pregnancy; however, the first ultrasound is usually performed during the first trimester to confirm the pregnancy. A second scan may be performed at about 18 to 20 weeks to look for congenital malformations. A third one may be done at around 34 weeks to evaluate fetal size and verify placental position.

When planning the care of a woman in the active phase of labor, the nurse would anticipate assessing the fetal heart rate at which interval? A) Every 2 to 4 hours B) Every 45 to 60 minutes C) Every 15 to 30 minutes D) Every 10 to 15 minutes

C. During the active phase of labor, FHR is monitored every 15 to 30 minutes.

A woman in labor who received an opioid for pain relief develops respiratory depression. The nurse would expect which agent to be administered? A) Butorphanol B) Fentanyl C) Naloxone D) Promethazine

C. Naloxone is an opioid antagonist used to reverse the effects of opioids such as respiratory depression.

When applying the ultrasound transducers for continuous external electronic fetal monitoring, at which location would the nurse place the transducer to record the FHR? A) Over the uterine fundus where contractions are most intense B) Above the umbilicus toward the right side of the diaphragm C) Between the umbilicus and the symphysis pubis D) Between the xiphoid process and umbilicus

C. - The ultrasound transducer is positioned on the maternal abdomen in the midline between the umbilicus and the symphysis pubis. - The tocotransducer is placed over the uterine fundus in the area of greatest contractility.

When palpating the fundus during a contraction, the nurse notes that it feels like a chin. The nurse interprets this finding as indicating which type of contraction? A) Intense B) Strong C) Moderate D) Mild

C. -A contraction that feels like the chin typically represents a moderate contraction. -A contraction described as feeling like the tip of the nose indicates a mild contraction. - A strong contraction feels like the forehead.

A nurse is assisting with the delivery of a newborn. The fetal head has just emerged. Which of the following would be done next? A) Suctioning of the mouth and nose B) Clamping of the umbilical cord C) Checking for the cord around the neck D) Drying of the newborn

C. Checking for the cord around the neck once the fetal head has emerged, the primary care provider explores the fetal neck to see if the umbilical cord is wrapped around it. If it is, the cord is slipped over the head to facilitate delivery. * Then the health care provider suctions the newborn's mouth first (because the newborn is an obligate nose breather) and then the nares with a bulb syringe to prevent aspiration of mucus, amniotic fluid, or meconium.

When assessing fetal heart rate, the nurse finds a heart rate of 175 bpm, accompanied by a decrease in variability and late decelerations. Which of the following would the nurse do next? A) Have the woman change her position. B) Administer oxygen. C) Notify the health care provider. D) Continue to monitor the pattern every 15 minutes.

C. Fetal tachycardia as evidenced by a fetal heart rate greater than 160 bpm accompanied by a decrease in variability and late decelerations is an ominous sign indicating the need for prompt intervention. The health care provider should be notified immediately and then measures should be instituted such as having the woman lie on her side and administering oxygen.

A nurse is providing care to a woman during the third stage of labor. Which of the following would alert the nurse that the placenta is separating? A) Boggy, soft uterus B) Uterus becoming discoid shaped C) Sudden gush of dark blood from the vagina D) Shortening of the umbilical cord

C. Sudden gush of dark blood from the vagina

A woman is admitted to the labor and birthing suite. Vaginal examination reveals that the presenting part is approximately 2 cm above the ischial spines. The nurse documents this finding as: A) +2 station B) 0 station C) -2 station D) Crowning

C. The ischial spines serve as landmarks and are designated as zero status. If the presenting part is palpated higher than the maternal ischial spines, a negative number is assigned. Therefore, the nurse would document the finding as -2 station.

A nurse is explaining the use of therapeutic touch as a pain relief measure during labor. Which of the following would the nurse include in the explanation? A) "This technique focuses on manipulating body tissues." B) "The technique requires focusing on a specific stimulus." C) "This technique redirects energy fields that lead to pain." D) "The technique involves light stroking of the abdomen with breathing."

C. "This technique redirects energy fields that lead to pain." -Therapeutic touch is an energy therapy and is based on the premise that the body contains energy fields that lead to either good or ill health and that the hands can be used to redirect the energy fields that lead to pain. - Attention focusing and imagery involve focusing on a specific stimulus. - Massage focuses on manipulating body tissues. - Effleurage involves light stroking of the abdomen in rhythm with breathing.

A nursing instructor is describing the various childbirth methods. Which of the following would the instructor include as part of the Lamaze method? A) Focus on the pleasurable sensations of childbirth B) Concentration on sensations while turning on to own bodies C) Interruption of the fear-tension-pain cycle D) Use of specific breathing and relaxation techniques

D

At the first prenatal visit of all clients who come to your clinic, appropriate blood screenings are obtained. You realize that a hemoglobin A1c above which level is concerning for diabetes and warrants further testing? A. 5.0% B. 6.0% C. 5.5% D. 6.5%

D A hemoglobin A1c level of at least 6.5% is concerning for overt diabetes and further testing should be conducted to ensure the client is not diabetic. If glucose testing is not diagnostic of overt diabetes, the woman should be tested for gestational diabetes from 24 to 28 weeks of gestation with a 75-gm oral glucose tolerance test.

When providing preconception care to a client, which medication would the nurse identify as being safe to continue during pregnancy? A) Warfarin B) Lithium C) Accutane D) Famotidine

D Famotidine is a category B drug that has been used frequently during pregnancy and does not appear to cause major birth defects or other fetal problems. Accutane and warfarin are category X drugs and should never be taken during pregnancy. Lithium is a category D drug with clear health risks for the fetus and

Encouraging routine prenatal visits is an important function for nurses to ensure the clients avoid complications or difficulties throughout the pregnancy and delivery. All clients should be screened for gestational diabetes at which time during the pregnancy? A. Between 20 and 24 weeks' gestation B. Between 15 and 19 weeks' gestation C. Between 29 and 32 weeks' gestation D. Between 24 and 28 weeks' gestation

D Screening for gestational diabetes is best done between 24 and 28 weeks' gestation, unless screening is warranted in the first trimester for high-risk reasons. If the initial screening is elevated, then further testing should be conducted to confirm the diagnosis.

After teaching a pregnant woman how to count fetal movements, the nurse determines that the teaching was successful when the client states which of the following? A) "I won't expect more than three movements to happen in an hour." B) "I'll do the counts while I'm sitting and watching my son's basketball game." C)"I'll do the count once a week on a morning that I'm not rushed for work." D) "I'll sit comfortably in a recliner or lie on my side when I do the counts."

D The client should perform the counts in a relaxed environment and a comfortable position, such as a semi-Fowler's or side-lying position. The woman needs to do fetal movement counts consistently, at approximately the same time each day. A woman should report a count of fewer than three fetal movements in an hour. A relaxed environment, a comfortable position, and consistency in performing the counts are important to identify changes.

A potential complication for the mother and fetus is Rh incompatibility; therefore assessment should include blood typing. If the mother is Rh negative, her antibody titer should be evaluated. If treatment with RhoGAM is indicated, when should it be given? A. At 28 weeks B. At 36 weeks C. Only at birth D. At 32 weeks

If indicated, RhoGAM should be given at 28 weeks for prophylaxis and again following birth if the infant is Rh+.

A client states, "I think my waters broke! I felt this gush of fluid between my legs." The nurse tests the fluid with Nitrazine paper and confirms membrane rupture if the paper turns: A) Yellow B) Olive green C) Pink D) Blue

D. Amniotic fluid is alkaline and turns Nitrazine paper blue. * Nitrazine paper that remains yellow to olive green suggests that the membranes are most likely intact.

A woman in labor has chosen to use hydrotherapy as a method of pain relief. Which statement by the woman would lead the nurse to suspect that the woman needs additional teaching? A) "The warmth and buoyancy of the water has a nice relaxing effect." B) "I can stay in the bath for as long as I feel comfortable." C) "My cervix should be dilated more than 5 cm before I try using this method." D) "The temperature of the water should be at least 105°F."

D. The water temperature should not exceed body temperature. Therefore, a temperature of 105° F would be too warm.

A group of nursing students are reviewing the various medications used for pain relief during labor. The students demonstrate understanding of the information when they identify which agent as the most commonly used opioid? A) Butorphanol B) Nalbuphine C) Fentanyl D) Meperidine

D. Meperidine

Sperm maturation and storage in the male reproductive system occur in the: Testes Vas deferens Epididymis Seminal vesicles

Epididymis The function of the epididymis is to store and mature sperm until ejaculation occurs. the testes manufacture sperm and send them to the epididymis for storage and continued maturation.

The nurse is assessing the laboring client to determine fetal oxygenation status. What indirect assessment method will the nurse likely use? A. External electronic fetal monitoring B. Fetal blood pH C. Fetal oxygen saturation D. Fetal position

External electronic fetal monitoring is the first option that we use

The predominant anterior pituitary hormones that orchestrate the menstrual cycle include: Thyroid-stimulating hormone (TSH) Follicle-stimulating hormone (FSH) Corticotropin-releasing hormone (CRH) Gonadotropin-releasing hormone (GnRH)

Follicle-stimulating hormone (FSH)

A nurse is completing the assessment of a woman admitted to the labor and birth suite.Which of the following would the nurse expect to include as part of the physical assessment? (Select all that apply.) A) Current pregnancy history B) Fundal height measurement C) Support system D) Estimated date of birth E) Membrane status F) Contraction pattern

Fundal height measurement Membrane status Contraction pattern As part of the admission physical assessment, the nurse would assess: - fundal height - membrane status and contractions * Current pregnancy history, support systems, and estimated date of birth would be obtained when collecting the maternal health history.

A pregnant client with a history of spinal injury is being prepared for a cesarean birth. Which method of anesthesia is to be administered to the client? A. Local infiltration B. Epidural block C. Regional anesthesia D. General anesthesia

General anesthesia is administered in emergency cesarean births

A nurse is required to obtain the fetal heart rate (FHR) for a pregnant client. If the presentation is cephalic, which maternal site should the nurse monitor to hear the FHR clearly?

Lower quadrant of the maternal abdomen

The nurse is preparing to teach a class to a group of middle-aged women regarding the most common vasomotor symptoms experienced during menopause and possible modalities of treatment available. Common vasomotor symptoms would include which of the following? Chronic fatigue and confusion Forgetfulness and irritability Night sweats and hot flashes Decrease in sexual response and appetite

Night sweats and hot flashes The correct response is night sweats and hot flashes because vasomotor symptoms of menopause are usually described as night sweats and hot flashes. Vasomotor symptoms are the most commonly reported menopausal symptoms by women. These symptoms are associated with estrogen deficiency. Frequently, these symptoms can be controlled by lifestyle changes.

A nurse caring for a pregnant client in labor observes that the fetal heart rate is below 110 per minute. Which interventions should the nurse perform?

O2 mask

A woman's amniotic fluid is noted to be cloudy. The nurse interprets this finding as?

Possible infection

You are preparing to release a client who underwent a percutaneous umbilical blood sampling earlier in the day. You remind the client about the signs to watch for that could indicate an infection and you remind the client for the need to: A.Sleep sitting up for one night B.Soak in a tub of warm water if cramping occurs C. Remain on bed rest for 48 hours D. Count fetal movements as instructed

Prior to discharge, instruct the woman to report signs of infection, an increase in contractions or a change in fetal activity level from normal. Reinforce the need to count fetal movements, and review the technique so she can assess them when she is discharged home. Bed rest for 48 hours would be recommended only if there were other concerns. The client needs to report any continued cramping, and sleeping sitting upright would not be indicated.

After teaching a group of students about fetal heart rate patterns, the instructor determines the need for additional teaching when the students identify which of the following as indicating normal fetal acid-base status? (Select all that apply.) A) Sinusoidal pattern B) Recurrent variable decelerations C) Fetal bradycardia D) Absence of late decelerations E) Moderate baseline variability

Sinusoidal pattern Recurrent variable decelerations Fetal bradycardia

Ans: C Kegel exercises help to improve pelvic floor tone, strengthen perineal muscles, and promote healing, ultimately helping to prevent urinary incontinence later in life. Kegel exercises have no effect on lochia, involution, or pain.

The nurse develops a teaching plan for a postpartum client and includes teaching about how to perform Kegel exercises. The nurse includes this information for which reason? A) Reduce lochia B) Promote uterine involution C) Improve pelvic floor tone D) Alleviate perineal pain

Ans: A During the taking-in phase, new mothers when interacting with their newborns spend time claiming the newborn and touching him or her, commonly identifying specific features in the newborn such as "he has my nose" or "his fingers are long like his father's." Independence in self-care and interest in caring for the newborn are typical of the taking-hold phase. Confidence in caring for the newborn is demonstrated during the letting-go phase.

The nurse interprets which of the following as evidence that a client is in the taking-in phase? A) Client states, "He has my eyes and nose." B) Client shows interest in caring for the newborn. C) Client performs self-care independently. D) Client confidently cares for the newborn.

Ans: D If the woman is not breast-feeding, relief measures for engorgement include wearing a tight supportive bra 24 hours daily, applying ice to her breasts for approximately 15 to 20 minutes every other hour, and not stimulating her breasts by squeezing or manually expressing milk. Warm showers enhance the let-down reflex and would be appropriate if the woman was breast-feeding. Limiting fluid intake is inappropriate. Fluid intake is important for all postpartum women, regardless of the feeding method chosen.

The nurse is developing a teaching plan for a client who has decided to bottle feed her newborn. Which of the following would the nurse include in the teaching plan to facilitate suppression of lactation? A) Encouraging the woman to manually express milk B) Suggesting that she take frequent warm showers to soothe her breasts C) Telling her to limit the amount of fluids that she drinks D) Instructing her to apply ice packs to both breasts every other hour

Ans: A By the end of 10 days, the fundus usually cannot be palpated because it has descended into the true pelvis.

The nurse is making a follow-up home visit to a woman who is 12 days postpartum. Which of the following would the nurse expect to find when assessing the client's fundus? A) Cannot be palpated B) 2 cm below the umbilicus C) 6 cm below the umbilicus D) 10 cm below the umbilicus

Ans: B The partner's statement reflects stage 2 (reality), which occurs when fathers or partners realize that their expectations in stage 1 are not realistic. Their feelings change from elation to sadness, ambivalence, jealousy, and frustration. Many wish to be more involved in the newborn's care and yet do not feel prepared to do so. New fathers or partners pass through stage 1 (expectations) with preconceptions about what home life will be like with a newborn. Many men may be unaware of the dramatic changes that can occur when this newborn comes home to live with them. In stage 3 (transition to mastery), the father or partner makes a conscious decision to take control and be at the center of his newborn's life regardless of his preparedness. Taking-hold is a stage of maternal adaptation.

The partner of a woman who has given birth to a healthy newborn says to the nurse, "I want to be involved, but I'm not sure that I'm able to care for such a little baby." The nurse interprets this as indicating which of the following stages? A) Expectations B) Reality C) Transition to mastery D) Taking-hold

Ans: C Women who have had cesarean births tend to have less flow because the uterine debris is removed manually along with delivery of the placenta.

When caring for a mother who has had a cesarean birth, the nurse would expect the client's lochia to be: A) Greater than after a vaginal delivery B) About the same as after a vaginal delivery C) Less than after a vaginal delivery D) Saturated with clots and mucus

Ans: C Tachycardia in the postpartum woman warrants further investigation. It may indicate hypovolemia, dehydration, or hemorrhage. Deep red, fleshy-smelling lochia is a normal finding 6 hours postpartum. Voiding in small amounts such as less than 150 cc would indicate a problem, but 350 cc would be appropriate. Profuse sweating also is normal during the postpartum period.

When the nurse is assessing a postpartum client approximately 6 hours after delivery, which finding would warrant further investigation? A) Deep red, fleshy-smelling lochia B) Voiding of 350 cc C) Heart rate of 120 beats/minute D) Profuse sweating

Ans: B Despite a decrease in blood volume after birth, hematocrit levels remain relatively stable and may even increase. An acute decrease is not an expected finding. The WBC count remains elevated for the first 4 to 6 days and clotting factors remain elevated for 2 to 3 weeks. Bradycardia (50 to 70 beats per minute) for the first two weeks reflects the decrease in cardiac output.

Which of the following would lead the nurse to suspect that a postpartum woman is experiencing a problem? A) Elevated white blood cell count B) Acute decrease in hematocrit C) Increased levels of clotting factors D) Pulse rate of 60 beats/minute

Ans: B Identifying imperfections would not be associated with engrossment. Engrossment is characterized by seven behaviors: visual awareness of the newborn, tactile awareness of the newborn, perception of the newborn as perfect, strong attraction to the newborn, awareness of distinct features of the newborn, extreme elation, and increased sense of self-esteem.

Which of the following would the nurse interpret as being least indicative of paternal engrossment? A) Demonstrating pleasure when touching or holding the newborn B) Identifying imperfections in the newborn's appearance C) Being able to distinguish his newborn from others in the nursery D) Showing feelings of pride with the birth of the newborn

___________________ describes the irregular variations or absence of fetal heart rate (FHR) due to erroneous causes on the fetal monitor record.

artifact

What event occurs during the proliferative phase of the menstrual cycle? Menstrual flow starts Endometrium thickens Ovulation occurs Progesterone secretion peaks

endometrium thickens Under the influence of estrogen, the uterine lining (endometrium) thickens and grows in preparation for the implantation of a conceptus if that occurs or to be sloughed off during the menses.

A nurse is caring for a pregnant client who is in the active phase of labor. At what interval should the nurse monitor the client's vital signs?

every 30 minutes

If the nitrazine test is inconclusive, an additional test, called the ________________ test, can be used to confirm rupture of membranes

fern

- Green fluid may indicate that the fetus has passed meconium secondary to:

hypoxia, prolonged pregnancy, cord compression, intrauterine growth restriction, maternal hypertension, diabetes, or chorioamnionitis; however, it is considered a normal occurrence if the fetus is in a breech presentation.

During an admission assessment of a client in labor, the nurse observes that there is no vaginal bleeding yet. What nursing intervention is appropriate in the absence of vaginal bleeding? A. Monitor vital signs B. Assess amount of cervical dilation C. Obtain urine speicmen for urinalysis D. Monitor hydration status

if vaginal bleeding is absent during admission assessment, nurse should perform vaginal examination to assess amount of cervical dilation

Amniotic fluid should be clear when the membranes rupture, either spontaneously or artificially through an amniotomy. - Cloudy or foul-smelling amniotic fluid indicates __________ .

infection

FHR is assessed every 30 to 60 minutes during the _________ phase of labor.

latent

Of all of the synthetic opioids ___________ is the most commonly used opioid for the management of pain during labor.

meperidine

Respiratory depression Opioids given close to the time of birth can cause central nervous system depression, including respiratory depression, in the newborn, necessitating the administration of ____________

naloxone.

The ovarian cycle describes the series of events associated with the development of the _____________ within the ovaries.

ovum or ova

Which hormone is produced in high levels to prepare the endometrium for implantation just after ovulation by the corpus luteum? Estrogen Prostaglandins Prolactin Progesterone

progesterone

Fetal _______________ as evidenced by a fetal heart rate greater than 160 bpm accompanied by a decrease in variability and late decelerations is an ominous sign indicating the need for prompt intervention.

tachycardia

Sperm cells and the male hormone testosterone are made in which of the following structures? Select all that apply. Vas deferens Penis Scrotum Ejaculatory ducts Prostate gland Testes Seminiferous tubules Bulbourethral glands

testes and seminiferous tubules. Sperm is produced in the seminiferous tubules of the testes.

When assessing a pregnant woman in her last trimester, which question would be most appropriate to use to gather information about weight gain and fluid retention? A) "What's your usual dietary intake for a typical day?" B) "What size maternity clothes are you wearing now?" C) "How puffy does your face look by the end of a day?" D) "How swollen do your ankles appear before you go to bed?

D

When assessing a woman at follow-up prenatal visits, the nurse would anticipate which of the following to be performed? A) Hemoglobin and hematocrit B) Urine for culture C) Fetal ultrasound D) Fundal height measurement

D

The nurse is teaching a health education class on male reproductive anatomy and asks the students to identify the site of sperm production. Which structure, if identified by the group, would indicate to the nurse that the teaching was successful? A)Testes B)Seminal vesicles C)Scrotum D)Prostate gland

A The testes are responsible for sperm production. The seminal vesicles produce nutrient seminal fluid. The scrotum surrounds and protects the testes. The prostate gland and the seminal vesicles produce fluid to nourish the sperm.

A nurse is examining a female client and tests the client's vaginal pH. Which finding would the nurse interpret as normal? A)4.5 B)7 C)8.5 D)10

A The vagina has an acidic environment; therefore a pH of 4.5 would indicate an acidic environment. A pH of 7 is considered neutral; a pH above 7 is considered alkaline.

A nurse is conducting a class for a group of teenage girls about female reproductive anatomy and physiology. Which of the following would the nurse include as an external female reproductive organ? Select all that apply. A)Mons pubis B)Labia C)Vagina D)Clitoris E)Uterus

A The external female reproductive organs collectively are called the vulva (which means "covering" in Latin). The vulva serves to protect the urethral and vaginal openings and is highly sensitive to touch to increase the female's pleasure during sexual arousal (Stables & Rankin, 2010). The structures that make up the vulva include the mons pubis, the labia majora and minora, the clitoris, the structures within the vestibule, and the perineum. The vagina and uterus are internal female reproductive organs.

When describing the ovarian cycle to a group of students, which phase would the instructor include? A)Luteal phase B)Proliferative phase C)Menstrual phase D)Secretory phase

A The ovarian cycle consists of three phases: the follicular phase, ovulation, and the luteal phase. The endometrial cycle includes the proliferative phase, menstrual phase, and secretory phase.

After teaching a group of students about the discomforts of pregnancy, the students demonstrate understanding of the information when they identify which as common during the first trimester? (Select all that apply.) A) Urinary frequency B) Breast tenderness C) Cravings D) Backache E) Leg cramps

A,B,C

A pregnant woman is scheduled to undergo percutaneous umbilical blood sampling. When discussing this test with the woman, the nurse reviews what can be evaluated with the specimens collected. Which of the following would the nurse include? (Select all that apply.) A) Rh incompatibility B) Fetal acid-base status C) Sex-linked disorders D) Enzyme deficiencies E) Coagulation studies

A,B,E

pregnant woman is flying across the country to visit her family. After teaching the woman about traveling during pregnancy, which statement indicates that the teaching was successful? A) "I'll sit in a window seat so I can focus on the sky to help relax me." B) "I won't drink too much fluid so I don't have to urinate so often." C) "I'll get up and walk around the airplane about every 2 hours." D) "I'll do some upper arm stretches while sitting in my seat."

C

Assessment of a pregnant woman reveals a pigmented line down the middle of her abdomen. The nurse documents this as which of the following? A) Linea nigra B) Striae gravidarum C) Melasma D) Vascular spiders

Ans: A Linea nigra refers to the darkened line of pigmentation down the middle of the abdomen in pregnant women. Striae gravidarum refers to stretch marks, irregular reddish streaks on the abdomen, breasts, and buttocks. Melasma refers to the increased pigmentation on the face, also known as the mask of pregnancy. Vascular spiders are small, spiderlike blood vessels that appear usually above the waist and on the neck, thorax, face, and arms.

In a client's seventh month of pregnancy, she reports feeling ìdizzy, like I'm going to pass out, when I lie down flat on my back.î The nurse integrates which of the following in to the explanation? A) Pressure of the gravid uterus on the vena cava B) A 50% increase in blood volume C) Physiologic anemia due to hemoglobin decrease D) Pressure of the presenting fetal part on the diaphragm

Ans: A The client is describing symptoms of supine hypotension syndrome, which occurs when the heavy gravid uterus falls back against the superior vena cava in the supine position. The vena cava is compressed, reducing venous return, cardiac output, and blood pressure, with increased orthostasis. The increased blood volume and physiologic anemia are unrelated to the client's symptoms. Pressure on the diaphragm would lead to dyspnea.

A pregnant woman undergoes maternal serum alpha-fetoprotein (MSAFP) testing at 16 to 18 weeks' gestation. Which of the following would the nurse suspect if the woman's level is decreased? A) Down syndrome B) Sickle-cell anemia C) Cardiac defects D) Open neural tube defect

Ans: A Feedback: Decreased levels might indicate Down syndrome or trisomy 18. Sickle cell anemia may be identified by chorionic villus sampling. MSAFP levels would be increased with cardiac defects, such as tetralogy of Fallot. A triple marker test would be used to determine an open neural tube defect.

A nurse is describing advances in genetics to a group of students. Which of the following would the nurse least likely include? A) Genetic diagnosis is now available as early as the second trimester. B) Genetic testing can identify presymptomatic conditions in children. C) Gene therapy can be used to repair missing genes with normal ones. D) Genetic agents may be used in the future to replace drugs.

Ans: A Feedback: Genetic diagnosis is now possible very early in pregnancy (see Evidence-Based Practice 10.1). Genetic testing can now identify presymptomatic conditions in children and adults. Gene therapy can be used to replace or repair defective or missing genes with normal ones. Gene therapy has been used for a variety of disorders, including cystic fibrosis, melanoma, diabetes, HIV, and hepatitis (Tamura, Kamuma, Nakazato, et al. 2010). The potential exists for creation of increased intelligence and size through genetic intervention. Recent research using gene therapy shows promise for the generation of insulin-producing cells to cure diabetes (Calne, Gan, & Lee 2010). In the future, genetic agents may replace drugs, general surgery may be replaced by gene surgery, and genetic intervention may replace radiation.

When describing genetic disorders to a group of childbearing couples, the nurse would identify which as an example of an autosomal dominant inheritance disorder? A) Huntington's disease B) Sickle cell disease C) Phenylketonuria D) Cystic fibrosis

Ans: A Feedback: Huntington's disease is an example of an autosomal dominant inheritance disorder. Sickle cell disease, phenylketonuria, and cystic fibrosis are examples of autosomal recessive inheritance disorders.

After teaching a group of students about fetal development, the instructor determines that the teaching was successful when the students identify which of the following as providing the barrier to other sperm after fertilization? A) Zona pellucida B) Zygote C) Cleavage D) Morula

Ans: A Feedback: The zona pellucida is the clear protein layer that acts as a barrier to other sperm once one sperm enters the ovum for fertilization. The zygote refers to the union of the nuclei of the ovum and sperm resulting in the diploid number of chromosomes. Cleavage is another term for mitosis. The morula is the result of four cleavages leading to 16 cells that appear as a solid ball of cells. The morula reaches the uterine cavity about 72 hours after fertilization.

A group of students are reviewing the signs of pregnancy. The students demonstrate understanding of the information when they identify which as presumptive signs? (Select all that apply. A) Amenorrhea B) Nausea C) Abdominal enlargement D) Braxton-Hicks contractions E) Fetal heart sounds

Ans: A, B Presumptive signs include amenorrhea, nausea, breast tenderness, urinary frequency and fatigue. Abdominal enlargement and Braxton-Hicks contractions are probable signs of pregnancy. Fetal heart sounds are a positive sign of pregnancy.

A group of students are reviewing information about genetic inheritance. The students demonstrate understanding of the information when they identify which of the following as an example of an autosomal recessive disorder? (Select all that apply.) A) Cystic fibrosis B) Phenylketonuria C) Tay-Sachs disease D) Polycystic kidney disease E) Achondroplasia

Ans: A, B, C Feedback: Examples of autosomal recessive disorders include cystic fibrosis, phenylketonuria, and Tay-Sachs disease. Polycystic kidney disease and achondroplasia are examples of autosomal dominant diseases.

A nurse is assessing a pregnant woman on a routine checkup. When assessing the woman's gastrointestinal tract, which of the following would the nurse expect to find? (Select all that apply. A) Hyperemic gums B) Increased peristalsis C) Complaints of bloating D) Heartburn E) Nausea

Ans: A, C, D, E Gastrointestinal system changes include hyperemic gums due to estrogen and increased proliferation of blood vessels and circulation to the mouth; slowed peristalsis; acid indigestion and heartburn; bloating and nausea and vomiting.

A nurse is discussing fetal development with a pregnant woman. The woman is 12 weeks pregnant and asks, "What's happening with my baby?" Which of the following would the nurse integrate into the response? (Select all that apply.) A) Continued sexual differentiation B) Eyebrows forming C) Startle reflex present D) Digestive system becoming active E) Lanugo present on the head

Ans: A, D Feedback: At 12 weeks, sexual differentiation continues and the digestive system shows activity. Eyebrows form and startle reflex is present between weeks 21 and 24. Lanugo on the head appears about weeks 13-16.

A woman just delivered a healthy term newborn. Upon assessing the umbilical cord, the nurse would identify which of the following as normal? (Select all that apply.) A) One vein B) Two veins C) One artery D) Two arteries E) One ligament F) Two ligaments

Ans: A, D Feedback: The normal umbilical cord contains one large vein and two small arteries.

A woman comes to the prenatal clinic suspecting that she is pregnant, and assessment reveals probable signs of pregnancy. Which of the following would be included as part of this assessment? (Select all that apply A) Positive pregnancy test B) Ultrasound visualization of the fetus C) Auscultation of a fetal heart beat D) Ballottement E) Absence of menstruation F) Softening of the cervix

Ans: A, D, F Probable signs of pregnancy include a positive pregnancy test, ballottement, and softening of the cervix (Goodell's sign). Ultrasound visualization of the fetus, auscultation of a fetal heart beat, and palpation of fetal movements are considered positive signs of pregnancy. Absence of menstruation is a presumptive sign of pregnancy.

A nurse strongly encourages a pregnant client to avoid eating swordfish and tilefish because these fish contain which of the following? A) Excess folic acid, which could increase the risk for neural tube defects B) Mercury, which could harm the developing fetus if eaten in large amounts C) Lactose, which leads to abdominal discomfort, gas, and diarrhea D) Low-quality protein that does not meet the woman's requirements

Ans: B Nearly all fish and shellfish contain traces of mercury and some contain higher levels of mercury that may harm the developing fetus if ingested by pregnant women in large amounts. Among these fish are shark, swordfish, king mackerel, and tilefish. Folic acid is found in dark green vegetables, baked beans, black-eyed peas, citrus fruits, peanuts, and liver. Folic acid supplements are needed to prevent neural tube defects. Women who are lactose intolerant experience abdominal discomfort, gas, and diarrhea if they ingest foods containing lactose. Fish and shellfish are an important part of a healthy diet because they contain high-quality proteins, are low in saturated fat, and contain omega-3 fatty acids.

Assessment of a pregnant woman reveals that she compulsively craves ice. The nurse documents this finding as which of the following? A) Quickening B) Pica C) Ballottement D) Linea nigra

Ans: B Pica refers to the compulsive ingestion of nonfood substances such as ice. Quickening refers to the mother's sensation of fetal movement. Ballottement refers to the feeling of rebound from a floating fetus when an examiner pushes against the woman's cervix during a pelvic examination. Linea nigra refers to the pigmented line that develops in the middle of the woman's abdomen.

A primiparous client is being seen in the clinic for her first prenatal visit. It is determined that she is 11 weeks pregnant. The nurse develops a teaching plan to educate the client about what she will most likely experience during this period. Which of the following would the nurse include? A) Ankle edema B) Urinary frequency C) Backache D) Hemorrhoids

Ans: B The client is in her first trimester and would most likely experience urinary frequency as the growing uterus presses on the bladder. Ankle edema, backache, and hemorrhoids would be more common during the later stages of pregnancy.

A pregnant woman comes to the clinic and tells the nurse that she has been having a whitish vaginal discharge. The nurse suspects vulvovaginal candidiasis, based on which assessment finding? A) Fever B) Vaginal itching C) Urinary frequency D) Incontinence

Ans: B Vaginal secretions become more acidic, white, and thick during pregnancy. Most women experience an increase in a whitish vaginal discharge, called leukorrhea. This is normal except when it is accompanied by itching and irritation, possibly suggesting Candida albicans, a monilial vaginitis, which is a very common occurrence in this glycogen-rich environment. Fever would suggest a more serious infection. Urinary frequency occurs commonly in the first trimester, disappears during the second trimester, and reappears during the third trimester. Incontinence would not be associated with a vulvovaginal candidiasis. Incontinence would require additional evaluation.

The nurse is discussing the insulin needs of a primiparous client with diabetes who has been using insulin for the past few years. The nurse informs the client that her insulin needs will increase during pregnancy based on the nurse's understanding that the placenta produces: A) hCG, which increases maternal glucose levels B) hPL, which deceases the effectiveness of insulin C) Estriol, which interferes with insulin crossing the placenta D) Relaxin, which decreases the amount of insulin produced

Ans: B hPL acts as an antagonist to insulin, so the mother must produce more insulin to overcome this resistance. If the mother has diabetes, then her insulin need would most likely increase to meet this demand. hCG does not affect insulin and glucose level. Estrogen, not estriol, is believed to oppose insulin. In addition, insulin does not cross the placenta. Relaxin is not associated with insulin resistance.

Assessment of a pregnant woman reveals oligohydramnios. The nurse would be alert for the development of which of the following? A) Maternal diabetes B) Placental insufficiency C) Neural tube defects D) Fetal gastrointestinal malformations

Ans: B Feedback: A deficiency of amniotic fluid, oligohydramnios, is associated with uteroplacental insufficiency and fetal renal abnormalities. Excess amniotic fluid is associated with maternal diabetes, neural tube defects, and malformations of the gastrointestinal tract and central nervous system.

After teaching a group of students about fetal development, the instructor determines that the teaching was successful when the students identify which of the following as essential for fetal lung development? A) Umbilical cord B) Amniotic fluid C) Placenta D) Trophoblasts

Ans: B Feedback: Amniotic fluid is essential for fetal growth and development, especially fetal lung development. The umbilical cord is the lifeline from the mother to the growing embryo. The placenta serves as the interface between the mother and developing fetus. It secretes hormones and supplies the fetus with nutrients and oxygen needed for growth. The trophoblasts differentiate into all the cells that form that placenta.

After the nurse describes fetal circulation to a pregnant woman, the woman asks why her fetus has a different circulation pattern than hers. In planning a response, the nurse integrates understanding of which of the following? A) Fetal blood is thicker than that of adults and needs different pathways. B) Fetal circulation carries highly oxygenated blood to vital areas first. C) Fetal blood has a higher oxygen saturation and circulates more slowly. D) Fetal heart rates are rapid and circulation time is double that of adults.

Ans: B Feedback: Fetal circulation functions to carry highly oxygenated blood to vital areas first while shunting it away from less vital ones. Fetal blood is not thicker than that of adults. Large volumes of oxygenated blood are not needed because the placenta essentially takes over the functions of the lung and liver during fetal life. Although fetal heart rates normally range from 120 to 160 beats per minute, circulation time is not doubled.

A woman is scheduled to undergo fetal nuchal translucency testing. Which of the following would the nurse include when describing this test? A) "A needle will be inserted directly into the fetus's umbilical vessel." B) "You'll have an intravaginal ultrasound to measure fluid in the fetus." C) "The doctor will take a sample of fluid from your bag of waters." D) "A small piece of tissue from the fetal part of the placenta is taken."

Ans: B Feedback: Fetal nuchal translucency testing involves an intravaginal ultrasound that measures fluid collection in the subcutaneous space between the skin and cervical spine of the fetus. Insertion of a needle into the fetus's umbilical vessel describes percutaneous umbilical blood sampling. Taking a sample of fluid from the amniotic sac (bag of waters) describes an amniocentesis. Obtaining a small tissue specimen from the fetal part of the placenta describes chorionic villus sampling.

A nursing instructor is preparing a teaching plan for a group of nursing students about the potential for misuse of genetic discoveries and advances. Which the following would the instructor most likely include? A) Gene replacement therapy for defective genes B) Individual risk profiling and confidentiality C) Greater emphasis on the causes of diseases D) Slower diagnosis of specific diseases

Ans: B Feedback: Individual risk profiling based on an individual's genetic makeup can raise issues related to privacy and confidentiality. Gene replacement therapy for defective genes and a greater emphasis on looking at the causes of disease are considered benefits associated with genetic advances. Rapid, more specific diagnosis of diseases would be possible.

A nurse is teaching a class on X-linked recessive disorders. Which of the following statements would the nurse most likely include? A) Males are typically carriers of the disorders. B) No male-to-male transmission occurs. C) Daughters are more commonly affected with the disorder. D) Both sons and daughters have a 50% risk of the disorder.

Ans: B Feedback: Most X-linked disorders demonstrate a recessive pattern of inheritance. Males are more affected than females. A male has only one X chromosome and all the genes on his X chromosome will be expressed, whereas a female will usually need both X chromosomes to carry the disease. There is no male-to-male transmission (since no X chromosome from the male is transmitted to male offspring), but any man who is affected will have carrier daughters. If a woman is a carrier, there is a 50% chance that her sons will be affected and a 50% chance that her daughters will be carriers.

The nurse is developing a presentation for a community group of young adults discussing fetal development and pregnancy. The nurse would identify that the sex of offspring is determined at the time of: A) Meiosis B) Fertilization C) Formation of morula D) Oogenesis

Ans: B Feedback: Sex determination occurs at the time of fertilization. Meiosis refers to cell division resulting in the formation of an ovum or sperm with half the number of chromosomes. The morula develops after a series of four cleavages following the formation of the zygote. Oogenesis refers to the development of a mature ovum, which has half the number of chromosomes.

When describing the structures involved in fetal circulation, the nursing instructor describes which structure as the opening between the right and left atrium? A) Ductus venosus B) Foramen ovale C) Ductus arteriosus D) Umbilical artery

Ans: B Feedback: The foramen ovale is the opening between the right and left atrium. The ductus venosus connects the umbilical vein to the inferior vena cava. The ductus arteriosus connects the main pulmonary artery to the aorta. The umbilical artery carries blood to the placenta.

During a prenatal class for a group of new mothers, the nurse is describing the hormones produced by the placenta. Which of the following would the nurse include? (Select all that apply.) A) Prolactin B) Estriol C) Relaxin D) Progestin E) Human chorionic somatomammotropin

Ans: B, C, D, E Feedback: Estriol, relaxin, progestin, and human chorionic somatomammotropin are secreted by the placenta. Prolactin is secreted after delivery for breast-feeding.

A nurse is assessing a child with Klinefelter's syndrome. Which of the following would the nurse expect to assess? (Select all that apply.) A) Gross mental retardation B) Long arms C) Profuse body hair D) Gynecomastia E) Enlarged testicles

Ans: B, D Feedback: Manifestations of Klinefelter's syndrome include mild mental retardation, small testicles, infertility, long arms and legs, gynecomastia, scant facial and body hair, and decreased libido.

A nurse is developing a teaching plan about nutrition for a group of pregnant women. Which of the following would the nurse include in the discussion? (Select all that apply. A) Keep weight gain to 15 lb B) Eat three meals with snacking C) Limit the use of salt in cooking D) Avoid using diuretics E) Participate in physical activity

Ans: B, D, E To promote optimal nutrition, the nurse would recommend gradual and steady weight gain based on the client's prepregnant weight, eating three meals with one or two snacks daily, not restricting the use of salt unless instructed to do so by the health care provider, avoiding the use of diuretics, and participating in reasonable physical activity daily.

After teaching a class on the stages of fetal development, the instructor determines that the teaching was successful when the students identify which of the following as a stage? (Select all that apply.) A) Placental B) Preembryonic C) Umbilical D) Embryonic E) Fetal

Ans: B, D, E Feedback: The three stages of fetal development are the preembryonic, embryonic, and fetal stage. Placental and umbilical are not stages of fetal development.

A gravida 2 para 1 client in the 10th week of her pregnancy says to the nurse, ìI've never urinated as often as I have for the past three weeks.î Which response would be most appropriate for the nurse to make? A) ìHaving to urinate so often is annoying. I suggest that you watch how much fluid you are drinking and limit it.î B) ìYou shouldn't be urinating this frequently now; it usually stops by the time you're eight weeks pregnant. Is there anything else bothering you?î C) ìBy the time you are 12 weeks pregnant, this frequent urination should no longer be a problem, but it is likely to return toward the end of your pregnancy.î D) ìWomen having their second child generally don't have frequent urination. Are you experiencing any burning sensations?î

Ans: C As the uterus grows, it presses on the urinary bladder, causing the increased frequency of urination during the first trimester. This complaint lessens during the second trimester only to reappear in the third trimester as the fetus begins to descend into the pelvis, causing pressure on the bladder.

A woman in her second trimester comes for a follow-up visit and says to the nurse, ìI feel like I'm on an emotional roller-coaster.î Which response by the nurse would be most appropriate? A) How often has this been happening to you? B) Maybe you need some medication to level things out. C) Mood swings are completely normal during pregnancy. D) Have you been experiencing any thoughts of harming yourself?

Ans: C Emotional lability is characteristic throughout most pregnancies. One moment a woman can feel great joy, and within a short time she can feel shock and disbelief. Frequently, pregnant women will start to cry without any apparent cause. Some women feel as though they are riding an emotional roller-coaster. These extremes in emotion can make it difficult for partners and family members to communicate with the pregnant woman without placing blame on themselves for their mood changes. Clear explanations about how common mood swings are during pregnancy are essential.

The nurse is assessing a pregnant woman in the second trimester. Which of the following tasks would indicate to the nurse that the client is incorporating the maternal role into her personality? A) The woman demonstrates concern for herself and her fetus as a unit. B) The client identifies what she must give up to assume her new role. C) The woman acknowledges the fetus as a separate entity within her. D) The client demonstrates unconditional acceptance without rejection.

Ans: C Incorporation of the maternal role into her personality indicates acceptance by the pregnant woman. In doing so, the woman becomes able to identify the fetus as a separate individual. Demonstrating concern for herself and her fetus as a unit is associated with introversion and more commonly occurs during the third trimester. Identification of what the mother must give up to assume the new role occurs during the first trimester. Demonstrating unconditional acceptance without rejection occurs during the third trimester.

A woman suspecting she is pregnant asks the nurse about which signs would confirm her pregnancy. The nurse would explain that which of the following would confirm the pregnancy? A) Absence of menstrual period B) Abdominal enlargement C) Palpable fetal movement D) Morning sickness

Ans: C Only positive signs of pregnancy would confirm a pregnancy. The positive signs of pregnancy confirm that a fetus is growing in the uterus. Visualizing the fetus by ultrasound, palpating for fetal movements, and hearing a fetal heartbeat are all signs that make the pregnancy a certainty. Absence of menstrual period and morning sickness are presumptive signs, which can be due to conditions other than pregnancy. Abdominal enlargement is a probable sign.

A woman is at 20 weeks' gestation. The nurse would expect to find the fundus at which of the following? A) Just above the symphysis pubis B) Mid-way between the pubis and umbilicus C) At the level of the umbilicus D) Mid-way between the umbilicus and xiphoid process

Ans: C The uterus, which starts as a pear-shaped organ, becomes ovoid as length increases over width. By 20 weeks' gestation, the fundus, or top of the uterus, is at the level of the umbilicus and measures 20 cm. A monthly measurement of the height of the top of the uterus in centimeters, which corresponds to the number of gestational weeks, is commonly used to date the pregnancy.

Which of the following changes in the musculoskeletal system would the nurse mention when teaching a group of pregnant women about the physiologic changes of pregnancy? A) Ligament tightening B) Decreased swayback C) Increased lordosis D) Joint contraction

Ans: C With pregnancy, the woman's center of gravity shifts forward, requiring a realignment of the spinal curvatures. There is an increase in the normal lumbosacral curve (lordosis). Ligaments of the sacroiliac joints and pubis symphysis soften and stretch. Increased swayback and an upper spine extension to compensate for the enlarging abdomen occur. Joint relaxation and increased mobility occur due to the influence of the hormones relaxin and progesterone.

During a vaginal exam, the nurse notes that the cervix has a bluish color. The nurse documents this finding as: A) Hegar's sign B) Goodell's sign C) Chadwick's sign D) Ortolani's sign

Ans: C Bluish coloration of the cervix is termed Chadwick's sign. Hegar's sign refers to the softening of the lower uterine segment or isthmus. Goodell's sign refers to the softening of the cervix. Ortolani's sign is a maneuver done to identify developmental dysplasia of the hip in infants.

When describing amniotic fluid to a pregnant woman, the nurse would include which of the following? A) "This fluid acts as transport mechanism for oxygen and nutrients." B) "The fluid is mostly protein to provide nourishment to your baby." C) "This fluid acts as a cushion to help to protect your baby from injury." D) "The amount of fluid remains fairly constant throughout the pregnancy."

Ans: C Feedback: Amniotic fluid protects the floating embryo and cushions the fetus from trauma. The placenta acts as a transport mechanism for oxygen and nutrients. Amniotic fluid is primarily water with some organic matter. Throughout pregnancy, amniotic fluid volume fluctuates.

While talking with a pregnant woman who has undergone genetic testing, the woman informs the nurse that her baby will be born with Down syndrome. The nurse understands that Down syndrome is an example of: A) Multifactorial inheritance B) X-linked recessive inheritance C) Trisomy numeric abnormality D) Chromosomal deletion

Ans: C Feedback: Down syndrome is an example of a chromosomal abnormality involving the number of chromosomes (trisomy numeric abnormality), in particular chromosome 21, in which the individual has three copies of that chromosome. Multifactorial inheritance gives rise to disorders such as cleft lip, congenital heart disease, neural tube defects, and pyloric stenosis. X-linked recessive inheritance is associated with disorders such as hemophilia. Chromosomal deletion is involved with disorders such as cri du chat syndrome.

Prenatal testing is used to assess for genetic risks and to identify genetic disorders. In explaining to a couple about an elevated alpha-fetoprotein screening test result, the nurse would discuss the need for: A) Special care needed for a Down syndrome infant B) A more specific determination of the acid-base status C) Further, more definitive evaluations to conclude anything D) Immediate termination of the pregnancy based on results

Ans: C Feedback: Increased maternal serum alpha fetoprotein levels may indicate a neural tube defect, Turner syndrome, tetralogy of Fallot, multiple gestation, omphalocele, gastroschisis, or hydrocephaly. Therefore, additional information and more specific determinations need to be done before any conclusion can be made. Down syndrome is associated with decreased maternal serum alpha fetoprotein levels. This type of testing provides no information about the acid-base status of the fetus. Immediate termination is not warranted; more information is needed.

The nurse is teaching a pregnant woman about recommended weight gain. The woman has a prepregnancy body mass index of 26. The nurse determines that the teaching was successful when the woman states that she should gain no more than which amount during pregnancy? A) 35 to 40 pounds B) 25 to 35 pounds C) 28 to 40 pounds D) 15 to 25 pounds

Ans: D A woman with a body mass index of 26 is considered overweight and should gain no more than 15 to 25 pounds during pregnancy. Women with a body mass index of 18.5 to 24.9 (considered healthy weight) should gain 25 to 35 pounds. A woman with a body mass index less than 18.5 should gain 28 to 40 pounds.

After teaching a group of students about female reproductive anatomy, the instructor determines that the teaching was successful when the students identify which of the following as the site of fertilization? A)Vagina B)Uterus C)Fallopian tubes D)Vestibule

C Fertilization occurs in the distal portion of the fallopian tubes. The lining of the uterus is shed with menstruation. The vagina connects the external genitalia to the uterus. The vestibule is an oval area enclosed by the labia minora laterally.

While talking with a woman in her third trimester, which behavior indicates to the nurse that the woman is learning to give of oneself? A) Showing concern for self and fetus as a unit B) Unconditionally accepting the pregnancy without rejection C) Longing to hold infant D) Questioning ability to become a good mother

Ans: D Learning to give of oneself would be demonstrated when the woman questions her ability to become a good mother to the infant. Showing concern for herself and fetus as a unit reflects the task of ensuring safe passage throughout pregnancy and birth. Unconditionally accepting the pregnancy reflects the task of seeking acceptance of the infant by others. Longing to hold the infant reflects the task of seeking acceptance of self in the maternal role to the infant.

The nurse teaches a primigravida client that lightening occurs about 2 weeks before the onset of labor. The mother will most likely experience which of the following at that time? A) Dysuria B) Dyspnea C) Constipation D) Urinary frequency

Ans: D Lightening refers to the descent of the fetal head into the pelvis and engagement. With this descent, pressure on the diaphragm decreases, easing breathing, but pressure on the bladder increases, leading to urinary frequency. Dysuria might indicate a urinary tract infection. Constipation may occur throughout pregnancy due to decreased peristalsis, but it is unrelated to lightening.

A nursing instructor is teaching a class to a group of students about pregnancy, insulin, and glucose. Which of the following would the instructor least likely include as opposing insulin? A) Prolactin B) Estrogen C) Progesterone D) Cortisol

Ans: D Prolactin, estrogen, and progesterone are all thought to oppose insulin. As a result, glucose is less likely to enter the mother's cells and is more likely to cross over the placenta to the fetus. After the first trimester, hPL from the placenta and steroids (cortisol) from the adrenal cortex act against insulin. hPL acts as an antagonist against maternal insulin, and thus more insulin must be secreted to counteract the increasing levels of hPL and cortisol during the last half of pregnancy.

A pregnant client in her second trimester has a hemoglobin level of 11 g/dL. The nurse interprets this as indicating which of the following? A) Iron-deficiency anemia B) A multiple gestation pregnancy C) Greater-than-expected weight gain D) Hemodilution of pregnancy

Ans: D Feedback: During pregnancy, the red blood cell count increases along with an increase in plasma volume. However, there is a greater increase in the plasma volume as a result of hormonal factors and sodium and water retention. Thus, the plasma increase exceeds the increase in RBCs, resulting in hemodilution of pregnancy, which is also called physiologic anemia of pregnancy. Changes in maternal iron levels would be more indicative of an iron-deficiency anemia. Although anemia may be present with a multiple gestation, an ultrasound would be a more reliable method of identifying it. Weight gain does not correlate with hemoglobin levels.

When teaching a pregnant client about the physiologic changes of pregnancy, the nurse reviews the effect of pregnancy on glucose metabolism. Which of the following would the nurse include as the underlying reason for the effect? A) Pancreatic function is affected by pregnancy. B) Glucose is utilized more rapidly during a pregnancy. C) The pregnant woman increases her dietary intake. D) Glucose moves through the placenta to assist the fetus.

Ans: D Feedback: The growing fetus has large needs for glucose, amino acids, and lipids, placing demands on maternal glucose stores. During the first half of pregnancy, much of the maternal glucose is diverted to the growing fetus. The pancreas continues to function during pregnancy. However, the placental hormones can affect maternal insulin levels. The demand for glucose by the fetus during pregnancy is high, but it is not necessarily used more rapidly. Placental hormones, not the woman's dietary intake, play a major role in glucose metabolism during pregnancy.

After teaching a pregnant woman about the hormones produced by the placenta, the nurse determines that the teaching was successful when the woman identifies which hormone produced as being the basis for pregnancy tests? A) Human placental lactogen (hPL) B) Estrogen (estriol) C) Progesterone (progestin) D) Human chorionic gonadotropin (hCG)

Ans: D Feedback: The placenta produces hCG, which is the basis for pregnancy tests. This hormone preserves the corpus luteum and its progesterone production so that the endometrial lining is maintained. Human placental lactogen modulates fetal and maternal metabolism and participates in the development of the breasts for lactation. Estrogen causes enlargement of the woman's breasts, uterus, and external genitalia and stimulates myometrial contractility. Progesterone maintains the endometrium.

After teaching a group of adolescent girls about female reproductive development, the nurse determines that teaching was successful when the girls state that menarche is defined as a woman's first: A)Sexual experience B)Full hormonal cycle C)Menstrual period D)Sign of breast development

C Menarche is defined as the establishment of menstruation. It does not refer to the woman's first sexual experience, full hormonal cycle, or sign of breast development.

During a nonstress test, when monitoring the fetal heart rate, the nurse notes that when the expectant mother reports fetal movement, the heart rate increases 15 beats or more above the baseline. The nurse interprets this as: A) Variable decelerations B) Fetal tachycardia C) A nonreactive pattern D) Reactive pattern

D

A pregnant woman has a rubella titer drawn on her first prenatal visit. The nurse explains that this test measures which of the following? A) Platelet level B) Rh status C) Immunity to German measles D) Red blood cell count

C

A pregnant woman in the 36th week of gestation complains that her feet are quite swollen at the end of the day. After careful assessment, the nurse determines that this is an expected finding at this stage of pregnancy. Which intervention would be most appropriate for the nurse to suggest? A) "Limit your intake of fluids." B) "Eliminate salt from your diet." C) "Try elevating your legs when you sit." D) "Wear Spandex-type full-length pants."

C

On the first prenatal visit, examination of the woman's internal genitalia reveals a bluish coloration of the cervix and vaginal mucosa. The nurse records this finding as: Hegar's sign B) Goodell's sign C) Chadwick's sign D) Homans' sign

C

When preparing a woman for an amniocentesis, the nurse would instruct her to do which of the following? A) Shower with an antiseptic scrub. B) Swallow the preprocedure sedative. C) Empty her bladder. D) Lie on her left side.

C

When describing perinatal education to a pregnant woman and her partner, the nurse emphasizes that the primary goal of these classes is to: A) Equip a couple with the knowledge to experience a pain-free childbirth B) Provide knowledge and skills to actively participate in birth and parenting C) Eliminate anxiety so that they can have an uncomplicated birth D) Empower the couple to totally control the birth process

B

Which of the following would the nurse include when teaching a pregnant woman about chorionic villus sampling? A) "The results should be available in about a week." B) "You'll have an ultrasound first and then the test." C) "Afterwards, you can resume your exercise program." D) "This test is very helpful for identifying spinal defects."

B

When describing the male sexual response to a group of students, the instructor determines that the teaching was successful when they identify emission as which of the following? A)Semen forced through the urethra to the outside B)Movement of sperm from the testes and fluid into the urethra C)Dilation of the penile arteries with increased blood flow to the tissues. D)Body's return to the physiologic nonstimulated state

B Emission refers to the movement of sperm from the testes and fluids from the accessory glands into the urethra, where it is mixed to form semen. As the urethra fills with semen, the base of the erect penis contracts, thus increasing pressure. This pressure forces the semen through the urethra to the outside (ejaculation). Dilation of the penile arteries with increased blood flow describes erection. The body's return to the physiologic nonstimulated state describes resolution.

When describing the menstrual cycle to a group of young women, the nurse explains that estrogen levels are highest during which phase of the endometrial cycle? A)Menstrual B)Proliferative C)Secretory D)Ischemic

B Estrogen levels are the highest during the proliferative phase of the endometrial cycle, when the endometrial glands enlarge in response to increasing amounts of estrogen. Progesterone is the predominant hormone of the secretory phase. Levels of estrogen and progesterone drop sharply during the ischemic phase and fall during the menstrual phase.

A nursing instructor is describing the hormones involved in the menstrual cycle to a group of nursing students. The instructor determines the teaching was successful when the students identify follicle-stimulating hormone as being secreted by which of the following? A)Hypothalamus B)Anterior pituitary gland C)Ovaries D)Corpus luteum

B Follicle-stimulating hormone and luteinizing hormone are secreted by the anterior pituitary gland. The hypothalamus secretes gonadotropin-releasing hormone. The ovaries secrete estrogen. The corpus luteum secretes progesterone.

A woman comes to the clinic for an evaluation. During the visit, the woman tells the nurse that her menstrual cycles have become irregular. "I've also been waking up at night feeling really hot and sweating." The nurse interprets these findings as which of the following? A)Menopause B)Perimenopause C)Climacteric D)Menarche

B Perimenopause is the time period occurring 2 to 8 years prior to menopause during which women may experience physical changes associated with decreasing estrogen levels, which may include vasomotor symptoms of hot flashes, irregular menstrual cycles, sleep disruptions, forgetfulness, irritability, mood disturbances, decreased vaginal lubrication, night sweats, fatigue, vaginal atrophy, and depression (Burbos & Morris, 2011). Vasomotor symptoms (hot flushes and night sweats) are the most common complaints for which women seek treatment. Menopause or climacteric is defined as 1 year without a menstrual period. Menarche refers to the onset of the first menses.

The nurse is assessing a 13-year-old girl who has had her first menses. Which of the following events would the nurse expect to have occurred first? A)Evidence of pubic hair B)Development of breast buds C)Onset of menses D)Growth spurt

B Pubertal events preceding the first menses have an orderly progression beginning with the development of breast buds, followed by the appearance of pubic hair, then axillary hair, then a growth spurt. Menses typically occurs about 2 years after the start of breast development.

A nurse is describing the structure and function of the reproductive system to an adolescent health class. The nurse describes the secretion of the seminal vesicles as which of the following? A)Mucus-like B)Alkaline C)Acidic D)Semen

B The paired seminal vesicles secrete an alkaline fluid that contains fructose and prostaglandins. The fructose supplies energy to the sperm on its journey to meet the ovum, and the prostaglandins assist in sperm mobility. The Cowper's glands secrete a mucus-like fluid. The vagina is an acidic environment. Semen refers to the sperm-containing fluid.

A woman comes to the clinic complaining of a vaginal discharge. The nurse suspects that the client has an infection. When gathering additional information, which of the following would the nurse be least likely to identify as placing the client at risk for an infection? A)Recent antibiotic therapy for an upper respiratory infection B)Last menstrual period about 5 days ago. C)Weekly douching D)Frequent use of feminine hygiene sprays.

B The vagina has an acidic environment, which protects it against ascending infections. Antibiotic therapy, douching, perineal hygiene sprays, and deodorants upset the acid balance within the vaginal environment and can predispose women to infections. Menstrual period about 5 days ago is not considered a risk factor.

The nurse is preparing an outline for a class on the physiology of the male sexual response. Which event would the nurse identify as occurring first? A)Sperm emission B)Penile vasodilation C)Psychological release D)Ejaculation

B With sexual stimulation, the arteries leading to the penis dilate and increase blood flow into erectile tissue. Blood accumulates, causing the penis to swell and elongate. Sperm emission (movement of sperm from the testes and fluid from the accessory glands) occurs with orgasm. Orgasm results in a pleasurable feeling of physiologic and psychological release. Ejaculation results in the discharge of semen from the urethra.

When describing the hormones involved in the menstrual cycle, a nurse identifies which hormone as responsible for initiating the cycle? A)Estrogen B)Luteinizing hormone C)Progesterone D)Prolactin

B With the initiation of the menstrual cycle, luteinizing hormone rises and stimulates the follicle to produce estrogen. As this hormone is produced by the follicle, estrogen levels rise inhibiting the output of LH. Ovulation occurs after an LH surge damages the estrogen-producing cells leading to a decline in estrogen. The LH surge results in the corpus luteum, which produces estrogen and progesterone. These two levels rise, suppressing LH. Lack of LH promotes degeneration of the corpus luteum, which then leads to a decline in estrogen and progesterone. The decline of ovarian hormones ends their negative effect on the secretion of LH, which is then secreted and the menstrual cycle begins again. Prolactin is the hormone responsible for breast milk production.

A group of nursing students are reviewing information about the male reproductive structures. The students demonstrate understanding of the information when they identify which of the following as accessory organs? (Select all that apply.) A)Testes B)Vas deferens C)Bulbourethral glands D)Prostate gland E)Penis

B,C,D The organs of the male reproductive system include the two testes (where sperm cells and testosterone are made), the penis, the scrotum, and the accessory organs (epididymis, vas deferens, seminal vesicles, ejaculatory duct, urethra, bulbourethral glands, and prostate gland).

A nurse is working with a pregnancy woman to schedule follow-up visits for her pregnancy. Which statement by the woman indicates that she understands the scheduling? A) "I need to make visits every 2 months until I'm 36 weeks pregnant." B) "Once I get to 28 weeks, I have to come twice a month." C) "From now until I'm 28 weeks, I'll be coming once a month." D) "I'll make sure to get a day off every 2 weeks to make my visits."

C

A nurse measures a pregnant woman's fundal height and finds it to be 28 cm. The nurse interprets this to indicate which of the following? A) 14 weeks' gestation B) 20 weeks' gestation C) 28 weeks' gestation D) 36 weeks' gestation

C

Which female reproductive tract structure would the nurse describe to a group of young women as containing rugae that enable it to dilate during labor and birth? A)Cervix B)Fallopian tube C)Vagina D)Vulva

C The vagina is a tubular, fibromuscular organ lined with mucous membrane that lies in a series of transverse folds called rugae. These rugae allow for extreme dilation of the canal during labor and birth. The cervix, the lower portion of the uterus, is composed of fibrous connective tissue that dilates during labor. The fallopian tube transports the ovum from the ovary to the uterus. The vulva is a collective term used to refer to the external female reproductive organs (mons pubis, labia majora and minora, clitoris, vestibular structures, and perineum).

A pregnant woman needs an update in her immunizations. Which of the following vaccinations would the nurse ensure that the woman receives? A) Measles B) Mumps C) Rubella D) Hepatitis B

D

After teaching a group of pregnant women about breast-feeding, the nurse determines that the teaching was successful when the group identifies which hormone as important for the production of breast milk after childbirth? A)Placental estrogen B)Progesterone C)Gonadotropin-releasing hormone D)Prolactin

D After childbirth and expulsion of the placenta, prolactin stimulates the production of milk. Placental estrogen and progesterone stimulate the development of the mammary glands during pregnancy. Gonadotropin-releasing hormone induces the release of follicle-stimulating hormone and luteinizing hormone to assist with ovulation.

The nurse is explaining the events that lead up to ovulation. Which hormone would the nurse identify as being primarily responsible for ovulation? A)Estrogen B)Progesterone C)Follicle-stimulating hormone D)Luteinizing hormone

D At ovulation, a mature follicle ruptures in response to a surge of luteinizing hormone. Estrogen is predominant at the end of the follicular phase, directly preceding ovulation. Progesterone peaks 5 to 7 days after ovulation. Follicle-stimulating hormone is highest during the first week of the follicular phase of the cycle.

A woman comes to the clinic complaining that she has little sexual desire. As part of the client's evaluation, the nurse would anticipate the need to evaluate which hormone level? A)Progesterone B)Estrogen C)Gonadotropin-releasing hormone D)Testosterone

D Testosterone is needed to stimulate sexual desire in women. Thus, an evaluation of this level would be done. Progesterone is often called the hormone of pregnancy because of its calming effect (reduction in uterine contractions) on the uterus, allowing pregnancy to be maintained. Estrogen is the predominant hormone at the end of the follicular phase. Gonadotropin-releasing hormone induces the release of FSH and LH to assist with ovulation.

The nurse is creating a diagram that illustrates the components of the male reproductive system. Which structure would be inappropriate for the nurse to include as an accessory gland? A)Seminal vesicles B)Prostate gland C)Cowper's glands D)Vas deferens

D The vas deferens is part of the ductal system of the male reproductive system responsible for transporting sperm from the epididymis. Accessory glands include the seminal vesicles, prostate gland, and Cowper's or bulbourethral glands.

Which glands are located on either side of the female urethra and secrete mucus to keep the opening moist and lubricated for urination? Cowper's Bartholin's Skene's Seminal

Skene's Skene's glands are located close to the urethral opening and secrete mucus and lubricate during urination and sexual intercourse.


Related study sets

unit five, civil liberties and civil rights

View Set

Lesson 4 Estructura 4.1 Present tense of ir Review

View Set

Unity 3 Biology 1. What is the function of the active site of an enzyme?

View Set

Immanuel Kant's Moral Philosophy

View Set